Dermatologic and Cosmetic Surgery Flashcards

1
Q

1- These lesions were biopsied and demonstrated a grenz zone and mixed inflammatory infiltrate composed of eosinophils, neutrophils, plasma cells and lymphocytes, with RBC extravasation. Which of the following treatment modality has shown efficacy in treating this condition?

A. Pulsed dye
B. Nd:YAG
C. Alexandrite
D. Intense pulsed light
E. Excimer

A

Correct choice: A. Pulsed dye

Explanation: The histopathology describes granuloma faciale: dense, often nodular infiltrate in the mid dermis with a Grenz zone and mixed infiltrate of eosinophils, neutrophils, plasma cells, mast cells, and lymphocytes, with extravasated RBCs. Vascular ectasia with fibrin deposition may be seen. There are several reports of treatment with pulsed dye laser.

How well did you know this?
1
Not at all
2
3
4
5
Perfectly
2
Q

2- Which combination of chemical peel therapies is appropriate for the treatment of epidermal melasma?

A. Phenol and trichloroacetic acid 20%
B. Trichloroacetic acid 35% and Jessner’s solution
C. Multiple coats of trichloroacetic acid 35%
D. Trichloroacetic acid 20% and Septisol
E. Ascorbic acid 5% and trichloroacetic acid 20%

A

Correct choice: E. Ascorbic acid 5% and trichloroacetic acid 20%

Explanation: Superficial chemical peels can be used in treatment of melasma. 5% ascorbic acid can reduce the activity of tyrosinase to treat melasma. Soliman et al compared the treatment of melasma using a 20% TCA peel alone vs a combination 20% TCA peel with topical 5% ascorbic acid. When comparing the TCA alone-treated patients to patients treated with the combination TCA and ascorbic acid peel, a significant reduction in pigmentation was noted in the combination peel patients. trichloroacetic acid 35% and Jessner’s solution combination was used for medium depth peeling for mild to moderate photoaging. Phenol is a deep chemical peel, and part of the Baker– Gordon formula. Septisol is a surfactant that reduces skin tension and is used in deep chemical peels (part of the Baker–Gordon formula). Multiple coats of TCA 20% as well as TCA 35% can essentially act like a medium depth peel. Multiple coats of TCA 35% can cause post inflammatory hyperpigmentation in darker skin types.

How well did you know this?
1
Not at all
2
3
4
5
Perfectly
3
Q

3- The following patient recently underwent submental injections of deoxycholic acid (Kybella). One day later, she noticed an inability of her right lip depressors to pull her lip downward and laterally, particularly when smiling. What is the next best step?

A. Watchful waiting
B. Inject the opposite side to achieve symmetry
C. Obtain an MRI
D. Inject intralesional corticosteroids
E. Administer non-steroidal anti-inflammatory agents

A

Correct choice: A. Watchful waiting

Explanation: Kybella (deoxycholic acid) is an FDA-approved treatment for injection into the submental area. Cases of marginal mandibular nerve injury manifesting as an asymmetric smile or facial muscle weakness were reported in 4% of subjects in clinical trials. All cases resolved spontaneously within a median of 44 days. It should not be injected into, or in close proximity to, the marginal mandibular branch of the facial nerve. Given the results of the clinical trials for deoxycholic acid, the best initial step in management is watchful waiting and close monitoring of the patient. If the patient does not improve in the next 1-2 months, additional investigations can be considered.

How well did you know this?
1
Not at all
2
3
4
5
Perfectly
4
Q

A- 35 year-old woman comes to your office for treatment of melasma. She has tried topical creams with minimal improvement and you recommend a chemical peel. Which of the following does require neutralization?

A. Jessner’s
B. TCA
C. Salicylic acid
D. Glycolic acid
E. Baker-Gordon peel

A

Correct choice: D. Glycolic acid

Explanation: Glycolic acid peels required neutralization with sodium bicarbonate. The others listed do not require neutralization. All alpha hydroxy acids need neutralization, including glycolic acid, lactic acid and mandelic acid. The remaining answer choices do not require neutralization.

How well did you know this?
1
Not at all
2
3
4
5
Perfectly
5
Q

5-What type of flap does this image represent?

A. A-to-T advancement flap
B. Island pedicle flap
C. Rotation flap
D. Transposition flap
E. Interpolation flap

A

Correct choice: B. Island pedicle flap

Explanation: This image shows the specialized advancement flap referred to as an island pedicle, or V-to-Y, flap. The “island” is when skin is completely incised on all 3 sides. Its rich vascular supply comes from subcutaneous/muscular pedicle. It provides exceptional flap viability and mobility.This image does not show the other listed flaps.

How well did you know this?
1
Not at all
2
3
4
5
Perfectly
6
Q
  1. A- patient comes in for endovenous laser ablation for varicose veins. You choose a 1320nm or 1450 nm laser. Which is the target chromophore for these lasers?

A. oxygenated hemoglobin
B. oxygen
C. water
D. melanin
E. deoxygenated hemoglobin

A

Correct choice: C. water

Explanation: 810–1064 nm laser (targetting more Hb), as well as 1320, 1440 and 1500 nm lasers (targetting more water) can be used in Endovenous Laser Ablation. The latter wavelengths target tissue water within the vein wall, causing fibrosis, results in a more controlled, and uniform heating. The absorption of water increases above 1000nm. Deoxygenated hemoglobin is the main chromophore of the 810- and 980-nm lasers. Oxygenated hemoglobin peaks in absorption between 400-600nm (vascular lasers). Melanin has a downward sloping absorption curve from 300-1000nm. Oxygen is not a target chromophore.

How well did you know this?
1
Not at all
2
3
4
5
Perfectly
7
Q

7-The most appropriate laser to initially treat a port-wine stain on an infant’s cheek would have which of the following wavelengths?

A. 488 nm
B. 510 nm
C. 585 nm
D. 694 nm
E. 755 nm

A

Correct choice: C. 585 nm

Explanation: The 585 nm pulsed dye laser (PDL) targets intravascular oxyhemoglobin and is considered the first-line treatment of choice for most benign vascular lesions, including port-wine stains. The original PDL had a wavelength of 577 nm which was later modified to 585 nm to achieve deeper penetration yet still maintain vascular specificity. The Argon laser can produce a wavelength of 488 nm, which predominantly targets melanin as the chromophore and thus can be used to treat pigmented lesions. The Argon laser was previously used for port-wine stains, but it had increased rate of scarring and has largely fallen out of use. It is important to note that certain pulsed dye lasers can operate at a shorter wavelength of 510 nm, which also targets melanin as the chromophore. The Ruby laser produces a wavelength of 694 nm, which targets melanin too. The Alexandrite laser produces a wavelength of 755 nm, thereby predominantly targeting melanin as well. The Alexandrite laser can be used for dark or PDL-resistant port-wine stains.

How well did you know this?
1
Not at all
2
3
4
5
Perfectly
8
Q
  1. A- 68-year-old male presents with a tattoo that he has had for >50 years that is a dark gray/black. What is the desired clinical endpoint when treating tattoos or pigmented lesions with Q-switched lasers?

A. Tissue whitening
B. Perifollicular edema
C. Purpura
D. Mottled pigmentation
E. Erythema

A

Correct choice: A. Tissue whitening

Explanation: Tissue whitening is the desired clinical endpoint when operating a Q-switched laser. Perifollicular edema, purpura, mottled pigmentation and erythema are not clinical endpoints when operating a Q-switched laser.

How well did you know this?
1
Not at all
2
3
4
5
Perfectly
9
Q

9-What is the mechanism of action of Onabotulinumtoxin-A?

A. Inhibits exocytosis of pre-synaptic epinephrine
B. Inhibits endocytosis of post-synaptic epinephrine
C. Inhibits exocytosis of pre-synaptic acetylcholine
D. Inhibits endocytosis of post-synaptic acetylcholine
E. Increases synaptic breakdown of acetylcholine

A

Correct choice: C. Inhibits exocytosis of pre-synaptic acetylcholine

Explanation: Botulinum toxin exerts its effect by cleaving key proteins required for nerve activation. First, the toxin binds specifically to nerves which use the neurotransmitter acetylcholine. Once bound to the nerve terminal, the neuron takes up the toxin into a vesicle. As the vesicle moves farther into the cell, it acidifies, activating a portion of the toxin which triggers it to push across the vesicle membrane and into the cell cytoplasm. Once inside the cytoplasm, the toxin cleaves SNARE proteins preventing the cell from releasing vesicles of acetylcholine. This stops nerve signaling, leading to paralysis.

How well did you know this?
1
Not at all
2
3
4
5
Perfectly
10
Q
  1. A- 60-year-old female presents with numerous facial lentigines due to photo damage. She would like to try cosmeceuticals for pigment lightening in addition to hydroxy acids. Which of the following cosmeceuticals is not a pigment lightener?

A. Green tea
B. Arbutin
C. Licorice extract
D. Vitamin C
E. Aleosin

A

Correct choice: A. Green tea

Explanation: Green tea is an antioxidant and contains antioxidant polyphenols: epicatechin, epicatechin-3-gallate, epigallocatechin, epigallocatechin-3-gallate.

B - Arbutin: Arbutin is a naturally occurring gluconopyranoside; decreased tyrosinase activity and may inhibit melanosomal maturation.
C- Licorice extract: Licorice extract contains liquiritin and isoliquertin, which disperse melanin, and glabridin, which inhibits tyrosinase.
D - Vitamin C: Vitamin C may interrupt melanogenesis by interacting with copper ions.
E - Aleosin: Aleosin is a naturally occurring hydroxymethylchromone; inhibits tyrosinase by competitive inhibition.

How well did you know this?
1
Not at all
2
3
4
5
Perfectly
11
Q

11- What is the best treatment option for the lesion below?

A. Excision with repair
B. Mohs with repair
C. 1064-nm Nd:YAG (long-pulsed)
D. Erbium:YAG
E. Radiation

A

Correct choice: C. 1064-nm Nd:YAG (long-pulsed)

Explanation: Long-pulsed 1064 nm Nd:YAG is the best treatment option of those listed for the benign lesion of venous lake of the lip. Excision, Mohs, and radiation are not indicated for benign lesions. Erbium:YAG is ill suited for the treatment of a vascular papule of the lip.

How well did you know this?
1
Not at all
2
3
4
5
Perfectly
12
Q

12- Botulinum toxin A cleaves which protein to exert its effects?

A. Synaptobrevin
B. Syntaxin
C. Acetylcholinesterase
D. SNAP-25
E. Synaptophysin

A

Correct choice: D. SNAP-25

Explanation: Choice D is correct. Botulinum toxins A, C, E catalyze cleavage of SNAP-25 protein. Botulinum toxins B, D, F, and G catalyze cleavage of synaptobrevin (or vesicle- associated membrane protein, VAMP [Choice 1]).
Syntaxins are a family of membrane integrated proteins participating in exocytosis. They are not targeted by botulinum toxins. Acetylcholinesterase cleaves acetylcholine and is not targeted by botulinum toxin. Synaphtophysin is a synaptic vesicle protein with uncertain function. It is often stained when searching for cells of neuroendocrine origin.

How well did you know this?
1
Not at all
2
3
4
5
Perfectly
13
Q

13- After treating this patient with Onabotulinumtoxin-A, they return to clinic complaining of their right upper eyelid drooping and intefering with their vision. What is the mechanism of action of the medicated eye drops which has been shown to be beneficial when this adverse effect occurs?

A. alpha agonist which results in contraction of the superior tarsal muscle
B. alpha agonist which results in relaxation of the superior tarsal muscle
C. alpha antagonist which results in contraction of the superior tarsal muscle
D. alpha antagonist which results in relaxation of the superior tarsal muscle
E. beta agonist which results in contraction of the superior tarsal muscle

A

Correct choice: A. alpha agonist which results in contraction of the superior tarsal muscle

Explanation: Eyelid ptosis is a known potential complication of glabellar treatment with neurotoxins, likely due to diffusion of medication into the orbit and affecting the levator palpebrae superioris muscle. Apraclonidine is an alpha-agonist which can improve the ptosis by causing contraction of the superior tarsal muscle (occasionally referred to as Müller’s muscle). The other answer choices do not describe the mechanism of action of apraclonidine.

How well did you know this?
1
Not at all
2
3
4
5
Perfectly
14
Q

14- Which of the following criteria carries the worst prognosis for a patient with a squamous cell carcinoma?

A. Size of tumor > 1 cm
B. Verrucous carcinoma subtype
C. Invasion of nerve 0.15 mm in diameter
D. Anatomic location
E. Immunosuppression

A

Correct choice: C. Invasion of nerve 0.15 mm in diameter

Explanation: Many factors contribute to the aggressive behavior of cutaneous squamous cell carcinoma. For example, tumors in immunosuppressed patients and tumors with the histologic subtypes desmoplastic and acantholytic are often biologically aggressive squamous cell carcinomas. Tumor diameter > 2 cm is the risk factor most highly associated with disease specific death. Perineural involvement of nerves > 0.1 mm is associated with increased nodal metastasis and increased mortality risk.
The AJCC most recent staging system, published October 2016, uses tumor diameter > 2 cm as the distinguishing factors between T1 and T2 tumors. Other high risk factors in the AJCC-8 staging system which upstage tumors to T3 include diametere >4cm, minor bone erosion, invasion of nerves >0.1 mm, or deep invasion below the subcutaneous fat. T4 is reserved for major bone involveent under this staging system. Alternatively, the Brigham and Women’s Hospital (BWH) staging systems contains a high risk T2b category requiring 2 or more of the following risk factors: tumor diameter > 2 cm, poorly differentiated histology, perineural invasion >0.1 mm, or tumor invasion beyond fat (exlcluding bony invasion which automatically upstages tumor to T3).
Tumors greater than 2 cm are higher risk. Verrucous carcinoma is a well-differentiated subtype of SCC with low metastatic potential. Anatomic location and immunosuppression host factors, while

important, do not portend as high a risk of nodal and metastatic potential as tumor characteristics in the AJCC/BWH staging systems.

How well did you know this?
1
Not at all
2
3
4
5
Perfectly
15
Q

15- Which tattoo pigment is the most difficult to treat?

A. Titanium dioxide
B. Cobalt aluminate
C. Carbon
D. Iron oxide
E. Chromium oxide

A

Correct choice: A. Titanium dioxide

Explanation: Tattoo ink removal by laser light works using complementary colors/wavelengths on the color wheel. Pigment usually absorbs light emitted of the opposite color on the wheel. For example, green tattoo pigment is best removed using a red laser (694 nm) and conversely red pigment is best removed with a green laser (532 nm). White ink (titanium dioxide) is composed of many different colors and, as such, reflects the colors of most laser light instead of absorbing them. This property makes white tattoos difficult to treat by most commercially available lasers.
The remainder of the answer choices are commonly treated with an appropriately selected laser wavelength.

Reference: Ho SG, Goh CL. Laser tattoo removal: a clinical update. J Cutan Aesthet Surg. 2015 Jan-Mar;8(1):9-15. PMID: 25949017

How well did you know this?
1
Not at all
2
3
4
5
Perfectly
16
Q

16- Which of the following is responsible for the sensory innervation of the majority of the dorsal foot?

A. Sural nerve
B. Superficial peroneal nerve
C. Deep peroneal nerve
D. Common peroneal nerve
E. Saphenous nerve

A

Correct choice: B. Superficial peroneal nerve

Explanation: The superficial peroneal nerve (superficial fibular nerve) is a mixed nerve that carries sensory information from the anterolateral aspect of the leg and the greater part of the dorsum of the foot (except for the first web space, which is innervated by the deep peroneal nerve). Entrapment of the superficial peroneal nerve is not uncommon and can present with pain in the sensory distribution of the nerve with or without paresthesia. The other remaining nerves are not responsible for the sensory innervation of the majority of the dorsal foot.

How well did you know this?
1
Not at all
2
3
4
5
Perfectly
17
Q

17- Which of the following peels is the best option for treatment of this condition?

A. Trichloroacetic acid 10-25%
B. Baker Gordon Peel
C. Three coats of Tricholoracetic acid 35%
D. Trichloroacetic acid 50%
E. Tricholoracetic acid 35% + jessner’s solution

A

Correct choice: A. Trichloroacetic acid 10-25%
Explanation: Indications for superficial chemical peels include melasma (shown in picture). This patient has darker skin. Superficial chemical peels can be used in all Fitzpatrick skin types. Tricholoracetic acid (TCA) 10-25%, Jessner’s solution, and Glycolic acid 20-70% are the best three superficial peel options for treatment of melasma. Baker Gordon Peel is a deep chemical peel, not for use in darker skin types. TCA 35% is a medium depth peel. One coat (not 3 coats) can be considered a superficial peel. Medium depth peels have a higher risk for postinflammatory hyperpigmentation in darker skin types. TCA 50% and TCA 35% + Jessner’s solution are medium depth peels.

How well did you know this?
1
Not at all
2
3
4
5
Perfectly
18
Q

18- Which of the following injectable fillers is most likely to produce a lip nodule?

A. Poly-L-lactic acid
B. Human collagen
C. Hyaluronic acid
D. Human fibroblasts
E. Calcium hydroxyl-apatite
c

A

Correct choice: E. Calcium hydroxyl-apatite

Explanation: Of the answer choices, calcium hydroxyl-apatite (Radiesse) is most likely to produce lip nodules and is thus viewed as contraindicated for lip injections.
The remaining answers are less likely to produce lip nodules.

How well did you know this?
1
Not at all
2
3
4
5
Perfectly
19
Q
  1. A- patient had sclerotherapy 90 minutes ago. She develops flushing, palpitations, sweating, and vomiting. Which sclerosant was used?

A. hypertonic saline
B. polidocanol
C. glycerine
D. sodium tetradecyl sulfate
E. sodium morrhuate

A

Correct choice: B. polidocanol

Explanation: Polidocanol can cause disulfiram like reactions. 1 mL of polidocanol contains 40.5 mg of ethanol, and patients taking disulfiram (Antabuse) should be warned about a possible alcohol– disulfiram reaction.

How well did you know this?
1
Not at all
2
3
4
5
Perfectly
20
Q

20- While performing Mohs micrographic surgery for this squamous cell carcinoma (shown in picture), you sever a nerve that lies superficially in the temple, just beneath the thin dermis and subcutaneous fat. The patient would have trouble performing which of the following?

A. Raising his eyebrows
B. Grimacing
C. Smiling
D. Closing his eyes
E. Chewing food

A

Correct choice: A. Raising his eyebrows

Explanation: Temporal nerve injury may cause permanent paralysis of the ipsilateral frontalis muscle, causing loss of horizontal forehead rhytides and descent of the brow on the affected side. The temporal branch of the facial nerve courses between the superficial and deep temporalis fascia, penetrating the underside of the frontalis muscle from its lateral edges. It lies in the subcutaneous fat overlying the SMAS of the temple and lateral forehead region. The greatest potential for nerve damage lies in the lateral regions of the face. In 15% of patients, there is a cross-innervation to the frontalis muscle by the more inferior zygomatic branch of the facial nerve. In the other 85% of patients, violating the temporal nerve results in motor denervation and the inability to raise a “droopy” eyebrow. Flattening of the forehead with diminished visibility of wrinkles and skin tension lines on the ipsilateral side is easily noticed. Over time, the inability to raise the eyebrow can lead to eyebrow and lid ptosis and upper visual field compromise as muscular disuse atrophy progresses.

B – The marginal mandibular branch of the facial nerve innervates the lip depressors. Injury can cause asymmetry when smiling or grimacing. C – The marginal mandibular branch of the facial nerve innervates the lip depressors. Injury can cause asymmetry when smiling or grimacing. D – The zygomatic branch of the facial nerve innervates the orbicularis oculi muscles. An injury can lead to inability to completely appose the upper and lower eyelids and subsequent corneal desiccation. E - The marginal mandibular branch of the facial nerve innervates the lip depressors. Injury can cause a compromise in mouth function, resulting in drooling.

How well did you know this?
1
Not at all
2
3
4
5
Perfectly
21
Q

21- These lesions have been present since birth. A laser with which wavelength is the treatment of choice?

A. 488nm
B. 510nm
C. 585nm
D. 755nm
E. 1320nm

A

Correct choice: C. 585nm

Explanation: The image depicts a capillary malformation (post-wine stain), which is most effectively treated by the pulsed dye laser (PDL) which has a wavelength setting of 585-595nm as this targets hemoglobin as its chromophore. Laser wavelengths of 488nm, 510nm, and 755nm taget melanin as the chromophore. Water is the chromophore for a laser operating at a wavelength of 1320nm.

How well did you know this?
1
Not at all
2
3
4
5
Perfectly
22
Q

22- According to the guidelines of the American Society for Dermatologic Surgery, the recommended maximal safe dosage of tumescent lidocaine totally by local anesthesia for a 110-kg female undergoing liposuction is approximately:

A. 4,950-6050 mg
B. 3,850-4,900 mg
C. 2,750-3,800 mg
D. 770 mg
E. 550 mg

A

Correct choice: A. 4,950-6050 mg

Explanation: Years of worldwide experience have shown that 55 mg/kg tumescent lidocaine for liposuction is remarkably safe. This dosage is safe most of the time. Multiple large surveys involving thousands of procedures have found no evidence of tumescent lidocaine toxicity at recommended dosage.

How well did you know this?
1
Not at all
2
3
4
5
Perfectly
23
Q
  1. A- 22 year old male patient is developing early androgenetic alopecia. He is seen for hair transplant consultation. How should you counsel this patient?

A. He can be scheduled for hair transplantation since he is over age 21
B. Patients younger than 25 are not preferred for hair transplantation
C. Finasteride can help because 5 alpha reductase activity is lower in areas of balding
D. Transplanted hairs are not permanent and hair loss will continue in the transplanted hairs as well
E. Patients with scarring alopecia cannot receive hair transplantation

A

Correct choice: B. Patients younger than 25 are not preferred for hair transplantation

Explanation: Patients younger than 25 are not recommended for hair transplantation. 5 alpha reductase activity is higher in areas of balding. Finasteride inhibits 5 alpha reductase type II and reverses miniaturization of hair follicles. Transplanted hairs are permanent as are the hairlines they create, which should be kept in mind at transplantation. Patients with scarring alopecia may benefit from hair transplant. However, they should have no evidence of inflammation for 6 months before the transplantation.

How well did you know this?
1
Not at all
2
3
4
5
Perfectly
24
Q

24-A patient had sclerotherapy and reports the injection was painful. Afterwards, she developed an ulcer over the injection site at the foot and ankle. Which sclerosant was most likely used?

A. hypertonic saline
B. polidocanol
C. glycerin
D. sodium tetradecyl sulfate
E. sodium morrhuate

A

Correct choice: A. hypertonic saline

Explanation: Hypertonic saline has the highest risk of cutaneous necrosis. The disadvantage of hypertonic saline is pain associated with injections and ulcerogenic potential. Often, anesthetic agents such as lidocaine are added to the mixture to minimize the discomfort involved, by decreasing the concentration of the saline and through the direct anesthetic effect. Hypertonic saline (10%) mixed with dextrose (25%) is another hyperosmolar agent that has been used in vein sclerosing. This agent has the advantages of low allergenicity and decreased pain compared with higher concentrations of plain hypertonic saline. However, this mixture is currently not FDA approved and is a relatively weak sclerosant compared to other options available. Ulceration can also occur with non-foamed sodium tetradecyl sulfate that is more than 1% in concentration.

How well did you know this?
1
Not at all
2
3
4
5
Perfectly
25
Q

25- What is the tissue temperature is required for destruction of basal or squamous cell carcinomas with cryotherapy?

A. -5° C
B. -15° C
C. -25° C
D. -35° C
E. -50° C

c

A

Correct choice: E. -50° C
Explanation: The temperature required for destruction of basal or squamous cell carcinoma is -50 degrees C. For melanocyte necrosis, -5 degrees C is required, and for keratinocyte treatment, -25 degrees C.
c

How well did you know this?
1
Not at all
2
3
4
5
Perfectly
26
Q

26- The pictured lesion will be best treated by a laser with which wavelength?

A. 1450nm.
B. 755nm
C. 694nm
D. 585nm
E. 510nm

A

Correct choice: D. 585nm

Explanation: The pictured lesion is a cherry angioma, which is a vascular lesion. Thus, it should be treated by a laser with a wavelength targeting hemoglobin as a chromophore. The Pulsed dye laser set at a 585nm wavelength is the best option listed.

The remaining answer choices do not target hemoglobin as a chromophore, so they would not be the best option in treating this cherry angioma:
1450nm-Diode laser (target chromophore: water) 755nm-Alexandrite laser (target chromophore: melanin) 694nm-Ruby laser (target chromophore: melanin)
510nm-short wavelength Pulsed dye laser (target chromophore: melanin)

How well did you know this?
1
Not at all
2
3
4
5
Perfectly
27
Q

27- Which of the following medications has demonstrated efficacy in the treatment of dermatofibrosarcoma protuberans (DFSP)?

A. Dabrafenib
B. Ipilimumab
C. Gefitinib
D. Erlotinib
E. Imatinib

A

Correct choice: E. Imatinib

Explanation: DFSP is a malignant neoplasm that most commonly presents as a slow-growing large nodule or plaque with multiple protuberances on the trunk of middle-aged adults. It is most often due to a reciprocal translocation of t(17;22)(q22;q13) resulting in fusion of collagen 1-alpha-1 and platelet-derived growth factor B (COL1A1-PDGFB) which serves as an oncogene. This is the basis of treatment with imatinib, a PDGF inhibitor.

Dabrafenib, a BRAF inhibitor, and Ipilimumab, a CTLA-4 inhibitor, are used in the treatment of metastatic melanoma. Gefitinib and Erlotinib are EGFR inhibitors that do not have efficacy in the treatment of DFSP.

How well did you know this?
1
Not at all
2
3
4
5
Perfectly
28
Q

28- True statements regarding skin cancer in organ transplant recipients include all of the following except:

A. 65 fold increase in development of SCC compared with general population
B. Kidney transplant patients are at the highest risk of developing cutaneous malignancies
C. Cutaneous malignancies develop 3-5 years after organ transplantation
D. Extent of tumor development related to degree of immunosuppression
E. Skin cancer is the most common cancer in transplant patients

A

Correct choice: B. Kidney transplant patients are at the highest risk of developing cutaneous malignancies

Explanation: Heart transplant patients are at the highest risk of cutaneous malignancies due to the extent of immunosuppresion they get.All the other answer choices represent true statements.

How well did you know this?
1
Not at all
2
3
4
5
Perfectly
29
Q

29- You have excised a large lipoma from a patient’s back, and would like to use an absorbable suture with the greatest tensile strength that lasts the longest. You look in your surgical supply closet, and choose which of the following:

A. Polydioxanone
B. Polyglactin 910
C. Poliglecaprone 25
D. Polyglycolic acid
E. Polypropylene

A

Correct choice: A. Polydioxanone

Explanation: Polydioxanone (PDS) is a polymer of paradioxanone. It is a monofilament, absorbable suture with 180 d absorption and low reactivity. The tensile strength is the greatest, and it lasts the

longest. The tensile strenght of PDS is 50% at 30 days. Polypropylene is not an absorbable suture and the other listed absorbale sutures do not have as a high a tensile strength as PDS. The tensile strenght of polyglactin is 50% at 21 days. The tensile strenght of polyglycolic acid is 20% at 21 days. The tensile strenght of Poliglecaprone 25 is 50% at 7 days.

How well did you know this?
1
Not at all
2
3
4
5
Perfectly
30
Q

30- This patient presented for a different lesion but you see this during your exam and perform a biopsy. If the pigmented lesion seen here is 0.9mm deep, what is the recommended margin for excision?

A. <0.5cm
B. 0.5cm
C. 1.0cm
D. 2.0cm
E. >2.0cm

A

Correct choice: C. 1.0cm

Explanation: The recommendation for a margin excision for in situ melanomas is 0.5cm (not including lentigo maligna forms).
Margin excision for a melanoma up to 1mm in thickness is 1.0cm. Thickness between 1.01-2mm requires a 1-2cm surgical margin, and >2cm is needed when the tumor thickness reaches 2mm.
The recommendation for a margin excision for in situ melanomas is 0.5cm (not including lentigo maligna forms). Margin excision for a melanoma up to 1mm in thickness is 1.0cm. Thickness between 1.01-2mm requires a 1-2cm surgical margin, and >2cm is needed when the tumor thickness reaches 2mm.

How well did you know this?
1
Not at all
2
3
4
5
Perfectly
31
Q

31- The nasolabial two stage flap depends on which artery for flap survival:

A. Supratrochlear artery
B. Superior labial artery
C. Angular artery
D. Lateral Nasal artery
E. Inferior labial artery

A

Correct choice: C. Angular artery

Explanation: Angular artery provides blood supply for the nasolabial transposition flap.
c

How well did you know this?
1
Not at all
2
3
4
5
Perfectly
32
Q

32- Which of the following statements best explains the principal of collimation with respect to lasers?

A. Emission of a well-defined wavelength (rather than a band of wavelengths)
B. Light waves traveling in phase, both in time and space
C. The parallel nature of a beam of light
D. Light that is amplified traveling back and forth between the laser’s mirrors
E. Perpendicular beams of light that increase scatter

A

Correct choice: C. The parallel nature of a beam of light

Explanation: Collimation refers to the parallel nature of light beams that make a laser focused on even small spot sizes. Monochromicity refers to a single wavelength, coherence refers to light traveling in phase. Light that is amplified traveling back and forth between the laser’s mirrors is how lasers operate. Perpendicular beams of light that increase scatter are not related to collimation.

How well did you know this?
1
Not at all
2
3
4
5
Perfectly
33
Q
  1. A- 63-year-old woman is seen for consultation for treatment of varicose veins in her bilateral legs. When discussing risks and benefits for treatment, you review the risks of having a rare anaphylactic reaction. Which of the following sclerosing agents has the highest risk for anaphylaxis?

A. Glycerin
B. Polidocanol
C. Sodium morrhuate
D. Hypertonic saline
E. Sodium tetradecyl sulfate

A

Correct choice: C. Sodium morrhuate

Explanation: The highest risk of anaphylaxis is with sodium morrhuate (Choice 3). Other side effects include pain, skin necrosis, and hyperpigmentation.
Glycerin (Choice 1) and hypertonic saline (Choice 4) alone do not carry any risk of anaphylaxis. There is a very rare risk of anaphylaxis with polidocanol (Choice 2) and sodium tetradecyl sulfate (Choice 5).

How well did you know this?
1
Not at all
2
3
4
5
Perfectly
34
Q

34- According to the information provided, the glasses shown provide protection for which laser type?

A. Pulsed dye 585 nm
B. Alexandrite 755 nm
C. Nd:YAG 1064 nm
D. Ruby 694 nm
E. Nd:YAG 650 nm

A

Correct choice: C. Nd:YAG 1064 nm

Explanation: Nd:YAG 1064nm. The ranges provided by the laser glasses indicate the wavelenghts the glasses protect for. It is important to note a crystal, such as Nd:YAG may produce more than one wavelenght.

Reference: Reference: Robinson. Surgery of the Skin. 2nd Ed. Chapter 33.

How well did you know this?
1
Not at all
2
3
4
5
Perfectly
35
Q

35- This patient underwent Mohs surgery with a bilobed transposition flap repair. The patient presents for a 2 month follow-up with the following result (indicated by the arrow). What is a common cause of this phenomenon?

A. Hematoma
B. Post-operative infection
C. Flap necrosis
D. Inadequate undermining
E. Undersizing of the flap

A

Correct choice: D. Inadequate undermining

Explanation: The arrow is pointing to an area of a trapdoor deformity which presents as pincushioning or a protuberant appearance of a flap. Commonly it presents 3-6 weeks post-op and is very common on the nose. It is likely due to circumferential contraction of the flap causing the flap to buckle. Ways to reduce this outcome is to undermine extensively under the entire flap and surrounding edges, properly size a flap (no oversize it or “stuff” it), square the edges of the flap as U-shaped flaps are more likely to buckle, and thin the flap appropriately.
Oversizing the flap rather than undersizing can lead to trapdoor/pin-cushioning. Hematoma, post-op infection and flap necrosis can all lead to poor outcomes of healing but not as likely to cause trapdoor deformity.

How well did you know this?
1
Not at all
2
3
4
5
Perfectly
36
Q

36- In the 60-degree z-plasty technique, the length of the scar length is increased by what percentage?

A. 25%
B. 35%
C. 50%
D. 75%
E. 90%

A

Correct choice: D. 75%

Explanation: A Z-plasty is a transposition flap used to treat wound contractures and scars. It is performed to decrease tension on a wound, change the orientation of a scar, and improve overall scar appearance. A 60-degree Z-plasty will yield a 75% increase in scar length and a 90-degree change in scar direction. A 45-degree Z-plasty will yield a 50% increase in scar length, and a 30- degree Z-plasty will yield a 25% increase in scar length.

How well did you know this?
1
Not at all
2
3
4
5
Perfectly
37
Q

37- What is the most appropriate choice for treating a basal cell carcinoma 6 mm in size at this location in a 48 year old patient who is otherwise healthy and who has only made one skin cancer to date?

A. Mohs micrographic surgery with repair
B. Radiation
C. Electrodessication and curettage
D. Excision with repair
E. Cryotherapy and curettage

A

Correct choice: A. Mohs micrographic surgery with repair

Explanation: Mohs micrographic surgery is the correct choice for the patient presented who has a 6 mm basal cell carcinoma located on the nasolabial fold in the region of the apical triangle. While the patient may elect not to proceed with therapy or choose a therapy with a lower cure rate compared to Mohs micrographic surgery, the most appropriate treatment option should be encouraged. Excision would offer the second highest cure rate for a 6 mm basal cell carcinoma at the apical triangle of the face, but is sub-optimal to Mohs micrographic surgery. Radiation should not be used for this basal cell carcinoma that can otherwise be surgically approached.
Electrodessication and curettage and cryotherapy and curettage do not offer appropriate cure rates for a basal cell carcinoma on the face of our patient.

How well did you know this?
1
Not at all
2
3
4
5
Perfectly
38
Q

38- Which of the following absorbable sutures has the highest tissue reactivity?

A. Chromic gut
B. Plain gut
C. Polyglycolic acid
D. Polydioxanone
E. Poliglecaprone

A

Correct choice: B. Plain gut

Explanation: Plain gut has the highest tissue reactivity amongst absorbable sutures. Chromic gut has high tissue reactivity, but it is less so than plain gut. The other answer choices have low tissue reactivity.

How well did you know this?
1
Not at all
2
3
4
5
Perfectly
39
Q

39- What structure is being lifted by the dissecting probe?

A. Angular artery
B. Transverse facial artery
C. Buccal nerve
D. Mental nerve
E. Superior labial artery

A

Correct choice: A. Angular artery
Explanation: The identified structure is the angular artery. After forming the superior labial branch, the facial artery becomes known as the angular artery.The pictured structure is wider caliber vessel, thus not a nerve. The superior labial artery and the transverse facial artery have lateral-medial orientations as opposed to the caudal-cephalad orientation of the angular artery/facial artery.

How well did you know this?
1
Not at all
2
3
4
5
Perfectly
40
Q

40- What is the best choice of a wavelength of a laser to treat the changes you see here?

A. 595 nm
B. 1,064 nm
C. 810 nm
D. 755 nm
E. 10,600 nm

A

Correct choice: E. 10,600 nm

Explanation: CO2 resurfacing and ablative lasers (10,600 nm) are the best for severe rhinophyma as is seen here. The other wavelengths represent lasers that will be less effective in treating the disease at this late stage.

How well did you know this?
1
Not at all
2
3
4
5
Perfectly
41
Q

41- Fast absorbing gut sutures are broken down in the skin by which process?

A. Hydrolysis
B. Proteolysis
C. Acetylcholinesterases
D. Cytochrome p-450
E. They are not broken down, and must be physically removed.

A

Correct choice: B. Proteolysis

Explanation: Fast absorbing gut is made up of a natural fiber. Natural fibers include those that are absorblable such as surgical gut, and those that are nonabsorbable such as silk. Absorbable natural sutures are broken down by proteolysis. Synthetic absorbable sutures are broken down by hydrolysis.

How well did you know this?
1
Not at all
2
3
4
5
Perfectly
42
Q

42- For a graft to be successful, what are the following stages it must go through (in order and according to approximate time intervals)?

A. Neovascularization, maturation, inosculation, imbibition
B. Imbibition, inosculation, neovascularization, maturation
C. Inosculation, imbibition, maturation, neovascularization
D. Imbibition, neovascularization, inosculation, maturation
E. Inosculation, neovascularization, imbibition, maturation

A

Correct choice: B. Imbibition, inosculation, neovascularization, maturation

Explanation: All successful grafts go through the following stages: 1. Imbibition – for the first 48 hours the graft is sustained by plasma from the recipient bed, hyperemia 1-3 days 2. Inosculation – on day 2-3 blood vessels in the graft establish connections with the wound bed 3. Neovascularization – ingrowth of new vessels into the graft occurs at approximately one week 4. Maturation – months post-grafting, sensory innervation occurs and the graft becomes paler

Reference: Reference: Dermatology In-Review. 2016-2017 ed. p512.

How well did you know this?
1
Not at all
2
3
4
5
Perfectly
43
Q

43- The instument number 3 is what?

A. Iris scissor
B. Mayo scissor
C. Gradel scissor
D. Suture scissor
E. Straight Metzenbaum scissor

A

Correct choice: A. Iris scissor

Explanation: Iris scissor. They have two sharp points and used for tissue dissection. Reference: Reference: Robinson. Surgery of the Skin. 2nd ed. Ch. 4

How well did you know this?
1
Not at all
2
3
4
5
Perfectly
44
Q

44- Post-operatively, one of your surgical patients notices numbness of the inferior two-thirds of the ear. Which nerve may have been injured?

A. Great auricular nerve
B. Lesser occipital nerve
C. Auriculotemporal nerve
D. Facial nerve
E. Glossopharyngeal nerve

A

Correct choice: A. Great auricular nerve

Explanation: The great auricular nerve(C2, C3) exits approximately 6.5 cm below the external auditory canal along the posterior border of the sternocleidomastoid muscle and extends superiorly. Injury leads to numbness of the inferior two-thirds of the ear and the adjacent cheek and neck.

How well did you know this?
1
Not at all
2
3
4
5
Perfectly
45
Q

45- This patient underwent Mohs surgery and presents for suture removal. The following photo is an example of which type of closure?

A. Split-thickness skin graft
B. Full-thickness skin graft

C. Composite graft
D. Transposition flap
E. Interpolation flap

A

Correct choice: B. Full-thickness skin graft

Explanation: This is an example of a full thickness skin graft. There is a circular graft on the ear, and a linear scar on the preauricular cheek indicating this as the donor site. Donor sites are commonly taken from the pre or post-auricular skin as it is easy to hide the scar in those locations. A split thickness graft is composed of epidermis and some dermis and is very thin. The donor site is usually left to heal by secondary intent and taken from a site that is easy to conceal, therefore there would not be a linear scar on the preauricular cheek. A composite graft typically includes 2 tissue types such as cartilage and skin. Although composite grafts can be use to repair ears to rebuild the structure/foundation, it would require cartilage to be taken from a donor site on the nose or ear, and the photo does not show any donor sites that would indicate this. In addition a very small defect, such as the one shown in this image, would likely not require cartilage. An interpolation or transposition flap would not have a linear scar at the preauricular cheek.

How well did you know this?
1
Not at all
2
3
4
5
Perfectly
46
Q

46- Six weeks following surgery, a scar is at which strength?

A. 5%
B. 15%
C. 40%
D. 70%
E. 80%

A

Correct choice: C. 40%
Explanation: A scar’s strength at 6 weeks is 40%. Scar strengths based on timeframe:
2 weeks- 5%
3 weeks- 15%
6 weeks- 40%
6 months- 70%
1 year- 80% (maximum strength of scar compared to original strength prior to injury/surgery)

How well did you know this?
1
Not at all
2
3
4
5
Perfectly
47
Q

47- Pigmentation defects represent abnormalities limited to the color of the skin and are frequently encountered in practice arising from inflammatory conditions, systemic disorders, or extrinsic effects such as sun exposure or treatment with dialysis. A patient presents to you with yellowish skin from undergoing hemodialysis for renal failure as well as underlying solar elastosis. The yellowish discoloration resulting from solar elastosis and treatment with chronic dialysis can be camouflaged by applying which of the following complementary-colored opaque cosmetic foundations?

A. Green
B. Purple
C. Brown
D. White
E. Yellow

A

Correct choice: B. Purple

Explanation: Pigmentation defects or discolorations can be camouflaged either by applying an opaque cosmetic that allows none of the abnormal underlying skin tones to be appreciated or by applying foundations of complementary colors. For example, red discoloration can be camouflaged by applying a green foundation, since green is the complementary color to red. The blending of the red skin with the green foundation yields a brown tone, which can be readily covered by a more conventional facial foundation. Furthermore, yellow skin tones can be blended with a complementary-colored purple foundation to also yield brown tones.

1 - Green: Red discoloration can be camouflaged by applying a green foundation, since green is the complementary color to red.
3 - Brown: Brown hyperpigmentation from skin conditions such as melasma, lentigines, and nevi can be camouflaged by using a white undercover foundation color.
4 - White: Hypopigmentation and depigmentation from skin conditions such as postinflammatory hypopigmentation, congenital pigmentary disorders and vitiligo can be cmouflaged by using a brown undercover foundation color.
5 - Yellow: Yellow facial color from conditions such as solar elastosis, chemotherapy and dialysis can be camouflaged by applying a purple undercover foundation color.

How well did you know this?
1
Not at all
2
3
4
5
Perfectly
48
Q
  1. A- patient undergoes Mohs surgery with a full-thickness skin graft repair. A bolster dressing was then sewn on over the graft. The bolster dressing would aim to promote healing of the graft during which phase?

A. Imbibition
B. Inosculation
C. Neovascularization
D. Maturation
E. Delayed phase

A

Correct choice: A. Imbibition

Explanation: Imbibition is the first stage of healing for skin grafts. It takes place during the first 48 hours where the graft is recieving nutrients by passive diffusion from the underlying wound bed. It is imperative that there is good connection/adherence of the flap with the wound bed for this to occur. This can be acheived with basting (tacking or quilting) sutures and a bolster. The bolster also provides stability and protection from trauma to the delicate graft.
Inosculation occurs after 2-3 days later, where the blood vessels in the graft start to establish connections to the underlying wound bed.
Neovascularization is the growth of new vessels into the graft between 4-7 days. Maturation or Reinvervation occurs months after the graft is placed where sensory innervation develops and the graft becomes paler. Delayed phase is not a known healing phase for grafts.

How well did you know this?
1
Not at all
2
3
4
5
Perfectly
49
Q

49-Which of the following is true regarding iodophor (Betadine) as an antiseptic?

A. Gram + coverage only
B. Gram - coverage only
C. Flamable
D. Ototoxic
E. Inactivated by blood

A

Correct choice: E. Inactivated by blood

Explanation: Iodophor (Betadine) is an antiseptic in the halogen class with moderate onset of activity that is sustained up to 1 hour. Spectrum includes gram+ and gram- organisms. It is absorbed through skin, must be dry to be effective, is tissue damaging, and inactivated by blood. It sometimes causes an allergic contact dermatitis.

How well did you know this?
1
Not at all
2
3
4
5
Perfectly
50
Q
  1. A- shave biopsy is performed on the helix and electrosurgery is used for hemostasis. Which form of electrosurgery is shown in the image, in which the electrode is spaced a few millimeters from the defect?

A. Electrocautery
B. Electrocoagulation
C. Electrodessication
D. Electrosection
E. Electrofulguration

A

Correct choice: E. Electrofulguration

Explanation: The correct answer is electrofulguration (Choice 5), in which there is superficial ablation where the tip of probe does not contact the skin. This method allows an electric spark to jump to the skin causing superficial epidermal carbonization, protecting the underlying tissue from heat. Choice 1 is incorrect as electrocautery is not true electrosurgery because there is no current running through the patient. Instead, electrocautery uses heat to achieve hemostasis. Electrocoagulation (Choice 2), electrodessication (Choice 3), electrosection (choice 4) involve the probe being in direct contact with the tissue.

How well did you know this?
1
Not at all
2
3
4
5
Perfectly
51
Q

51- The patient declines surgical intervention for this cutaneous neoplasm and a decision is made to pursue medical therapy. What is the most common side effect of the FDA-approved medication for this neoplasm

A. Nausea
B. Flu-like illness
C. Chest pain
D. Muscle spasms
E. Alopecia

A

Correct choice: D. Muscle spasms

Explanation: Vismodegib is smoothened (SMO) inhibitor approved for the treatment of adults with basal cell nevus syndrome, metastatic or locally advanced BCC that has recurred following surgery or who are not candidates for surgery or radiation. The most commonly reported side effect is muscle spasm, followed by alopecia and dysgeusia.

How well did you know this?
1
Not at all
2
3
4
5
Perfectly
52
Q

52- Which of the following sites would be appropriate for secondary intention healing?

A. Medial canthus
B. Alar crease
C. Conchal bowl
D. Lower eyelid
E. A, B, & C

A

Correct choice: E. A, B, & C

Explanation: Secondary intention healing occurs when the wound is allowed to heal on its own. This process allows for healing primarily by the contraction of myofibroblasts. It may be an excellent alternative for select defects, ie. surgical defects on concave areas, such as the medial canthus, alar crease, temple (as long as the defect is not too close to a free margin or anatomical landmark), and conchal bowl (as long as defect does not approach the external auditory meatus).
The remaining answer choices are incorrect.

How well did you know this?
1
Not at all
2
3
4
5
Perfectly
53
Q

53- Which of the following wound care dressings is the most appropriate choice for a dry, painful wound?

A. Foams
B. Hydrogels
C. Hydrofilms
D. Hydrocolloids
E. Alginates

A

Correct choice: B. Hydrogels

Explanation: Hydrogels are good for dry painful wounds. Foams are absorptive. Hydrofilms are occlusive, allowing gas and water vapor to permeate. Hydrocolloids are fibrinolytic, cause angiogenesis, inhibit keratinocyte migration, antibacterial increases healing rate, can cause surrounding maceration. Alginates are the most absorptive and are turned into gel by the wound exudate.

How well did you know this?
1
Not at all
2
3
4
5
Perfectly
54
Q

54- Which of the following is the most effective topical treatment option for pulsed dye laser- induced purpura?

A. Vitamin A
B. Vitamin C
C. Vitamin D
D. Vitamin E
E. Vitamin K

A

Correct choice: E. Vitamin K

Explanation: Vitamin K oxide gel appears to hasten the resolution of pulsed dye laser- induced purpura in subjects being treated for bilateral facial telangiectasia, and may well be useful in accelerating resolution of facial bruising from other cosmetic procedures such as fillers used for soft-tissue augmentation as well as other types of cutaneous surgical procedures.

How well did you know this?
1
Not at all
2
3
4
5
Perfectly
55
Q

55- All of the following are major antioxidants in the human epidermis except?

A. Superoxide dismutase
B. Coenzyme Q10
C. L-Ascorbate
D. Cathelicidin
E. Alpha-tocopherol

A

Correct choice: D. Cathelicidin

Explanation: Cathelicidin is an anti-microbial peptide in human epidermis thought to be upregulated in conditions such as psoriasis and rosacea. The remainder of the answers are all antioxidants which are naturally found in the skin. Research into the preventive role of topical antioxidants in photoaging is based on the free radical theory of aging. Epidermal depletion of these substances has been shown to be an early and sensitive marker of environmental oxidative damage.

How well did you know this?
1
Not at all
2
3
4
5
Perfectly
56
Q
    • A patient comes in for endovenous laser ablation for varicose veins. Which type of anesthesia should be used?

A. general anesthesia
B. tumescent anesthesia
C. 1% lidocaine without epinephrine
D. 0.05% marcaine with epinephrine
E. 1% sodium tetradecyl sulfate

A

Correct choice: B. tumescent anesthesia

Explanation: Tumescent anesthesia is used for endovenous radiofrequency ablation and endovenous laser ablation. This allows separation of the vessel from the surrounding tissue, thereby protecting surrounding tissue (including nerves) from injury. Endovenous ablation procedures can be done in an outpatient setting, without general anesthesia. 1% sodium tetradecyl sulfate is a sclerosant, not an anesthetic.

How well did you know this?
1
Not at all
2
3
4
5
Perfectly
57
Q

57-Which of the following best describes a tip stitch?

A. Half-buried horizontal mattress suture
B. Modified vertical mattress suture
C. Running horizontal mattress suture
D. Buried vertical mattress suture
E. Purse-string suture

A

Correct choice: A. Half-buried horizontal mattress suture

Explanation: A tip stitch is a half-buried horizontal mattress suture. It is most commonly used to align tissue and secure the tips of flaps. A pulley suture is a modified vertical mattress suture and is used for areas of high tension. The other options are incorrect.

How well did you know this?
1
Not at all
2
3
4
5
Perfectly
58
Q
  1. A- patient is scheduled for an excision on the pretibial area. Choose the most appropriate dermal suture for this site:

A. Polydioxanone
B. Poliglecaprone 25
C. Chromic gut
D. Polyester
E. Glycomer 631

A

Correct choice: A. Polydioxanone

Explanation: Polydioxanone (PDS) is an absorbable suture that lasts 180 days and is good for high tension areas such as the leg. Monofilament sutures such as poliglecaprone 25 and glycomer 631 work, but not as well in terms of duration of tensile strength for high tension areas.

How well did you know this?
1
Not at all
2
3
4
5
Perfectly
59
Q

59-A 62 year-old man presented for Mohs surgery for a basal cell carcinoma at his right posterior ear. The tumor was cleared after 2 stages, and was repaired as seen in the photo. What is the first phase of full thickness graft healing?

A. Imbibition
B. Inosculation
C. Neovascularization
D. Epithelial proliferation
E. Maturation

A

Correct choice: A. Imbibition

Explanation: A full thickness skin graft was used here to repair the post-surgical defect. A full thickness skin graft consists of the entire epidermis and dermis, the subcutaneous tissue removed prior to inset to avoid graft compromise. All successful grafts go through the following stages: imbibition (first 48 hours), inosculation (day 2-3), neovascularization (~1 week), and maturation (months post-grafting).

How well did you know this?
1
Not at all
2
3
4
5
Perfectly
60
Q

60-A 65-year-old man presented for Mohs micrographic surgery of a basal cell carcinoma at his left posterior ear. You inject local anesthetic in a ring block fashion to provide adequate anesthesia for the surgical procedure. At which anatomic location of the ear will cutaneous sensation remain intact?

A. Auricle
B. Helix
C. Conchal bowl
D. Tragus
E. Antihelix

A

Correct choice: C. Conchal bowl

Explanation: A ring block around the ear anesthetizes the entire ear, except for the conchal bowl and external auditory meatus. The vagus nerve provides the cutaneous sensation for the conchal bowl.
The great auricular nerve innervates the majority of the posterior ear, as well as 3/4 of the anterior helix. The auriculotemporal nerve innervates the tragus, anterior helix and part of the superior helix.

How well did you know this?
1
Not at all
2
3
4
5
Perfectly
61
Q

61-A surgical patient calls you post-operatively, after noticing that he can no longer abduct his arm after his Mohs procedure today. Which nerve may have been injured in this case?

A. C2 and C3
B. Supraclavicular nerve
C. CN XI
D. CN VII
E. CN IX

A

Correct choice: C. CN XI
Explanation: Injury of the spinal accessory nerve results in the inability to elevate the shoulder on the affected side, winged scapula, and the inability to initiate arm abduction.

How well did you know this?
1
Not at all
2
3
4
5
Perfectly
62
Q

62- The laser glasses shown in this photo have the letters “O.D.” printed on them. What does O.D. stand for?

A. A logarithmic measure of the power transmission factor
B. The appropriate wavelength of light for these glasses
C. The antireflection coating to suppress unwanted reflections
D. The appropriate beam diameter for these glasses
E. A measure of light as waves with fixed phases in both time and space

A

Correct choice: A. A logarithmic measure of the power transmission factor

Explanation: The O.D. stands for optical density, identifying the amount of the medium that may penetrate a protection filter. This is a logarithmic measure: for example, O.D. of 4 means that the optical power is attenuated by the factor 10 to the power of 4= 10000 times. E is the definition of coherence. Reference: https://safety.vanderbilt.edu/laser/glossary-laser-terms.php https://www.rp- photonics.com/optical_density.html

How well did you know this?
1
Not at all
2
3
4
5
Perfectly
63
Q

63- An obese patient comes in for liposuction consultation and indicates several areas she would like treated on the abdomen, back, hips, and thighs. She thinks this will assist in weight loss and her appearance. How should you counsel this patient?”

A. Liposuction will help in weight reduction
B. There is no limit to the amount of fat that can be removed in a session
C. Moderate to severe obesity are contraindications to liposuction
D. Liposuction can help improve the cellulite along the hips, outer thighs, buttocks
E. Because of her size, the procedure will need to be done under general anesthesia

A

Correct choice: C. Moderate to severe obesity are contraindications to liposuction

Explanation: Liposuction is not intended for weight reduction. Moderate obesity is relative contraindication, while severe obesity is an absolute contraindication. The upper limit of fat extraction in one setting is 4500-5000mL, according to American Academy of Dermatology guidelines. Beyond this, fluid shifts can occur, resulting in cardiovascular compromise. Liposuction does not improve the appearance of cellulite. Tumescent anesthesia in the office for liposuction has a high safety profile. The serious complications seen in liposuction are associated with general anesthesia, not with procedures performed with local tumescent anesthesia. Although deaths have been reported during liposuction, none has occurred when patients were treated with tumescent anesthesia alone. Office-based tumescent liposuction performed by dermatologic surgeons is safe and has a lower complication rate than hospital-based procedures.

How well did you know this?
1
Not at all
2
3
4
5
Perfectly
64
Q

64- You diagnose a 0.35 mm Breslow depth melanoma on the right shoulder of a 62 year old male. What is the appropriate and necessary medical treatment?

A. Reassurance
B. Excision with 5 mm margin and sentinel lymph node biopsy
C. Radiation therapy
D. Excision with 10 mm margin
E. Excision with 15 mm margin and sentinel lymph node biopsy
c

A

Correct choice: D. Excision with 10 mm margin

Explanation: The correct approach is excision with 10 mm (1 cm) margin with repair.

5 mm and 15 mm are not the appropriate margins for excision of a 0.35 mm BD melanoma and sentinel lymph node biopsy is not indicated. Radiation therapy is not the appropriate treatment.

How well did you know this?
1
Not at all
2
3
4
5
Perfectly
65
Q

65- Which of the following is FALSE regarding Mohs micrographic surgery?

A. Most large studies show a complication rate of below 1% to 3%
B. No deaths have been reported as a complication of Mohs surgery
C. Infection rates have been shown to be unaffected by use of nonsterile vs sterile gloves
D. Wrong site surgery is the most common reason for legal claims against Mohs surgeons
E. With regard to biopsy site identification prior to surgery, photographic documentation does not improve accuracy compared to clinical notes and diagrams

A

Correct choice: E. With regard to biopsy site identification prior to surgery, photographic documentation does not improve accuracy compared to clinical notes and diagrams

Explanation: The correct answer is E (E is false). Photographic documentation has shown to improve biopsy site identification prior to Mohs surgery [ii. Hansen TJ, Lolis M, Goldberg DJ, MacFarlane DF. Patient safety in dermatologic surgery: Part I. Safety related to surgical procedures. J Am Acad Dermatol. 2015 Jul;73(1):1-12. ]

How well did you know this?
1
Not at all
2
3
4
5
Perfectly
66
Q
  1. A- 70 year-old woman presents for follow-up 1 week after Mohs micrographic surgery of a poorly differentiated squamous cell carcinoma at the left temple. She is noted to have flattening of the left forehead and inability to raise her left eyebrow. At which anatomic location did the suspected nerve injury occur?

A. 2 cm posterior to lateral oral commissure
B. At the lateral canthus
C. 3 cm above the left eyebrow
D. Between the lateral eyebrow and anterior hairline
E. Anterior hairline at mid-pupillary level

A

Correct choice: D. Between the lateral eyebrow and anterior hairline

Explanation: The temporal nerve runs 0.5cm below the tragus to 1.5cm above the lateral brow. The danger zone of the temporal branch of CN VII occurs within a 2 cm box from lateral eyebrow to anterior hairline where it runs beneath the SMAS. Injury to this nerve results in unilateral forehead paralysis, ipsilateral forehead flattening, and eyelid ptosis.

How well did you know this?
1
Not at all
2
3
4
5
Perfectly
67
Q

67-A patient is scheduled for liposuction on the hips, buttocks, knees, arms, and back. In which body part can compartment syndrome occur after liposuction?

A. hips
B. buttocks
C. knees
D. arms
E. back

A

Correct choice: D. arms

Explanation: Compartment syndrome can occur in the arms, distal to the area infused with tumescent anesthesia due to fluid compression. Avoid anterior flexural surfaces. Postliposuction edema can occur in the thighs and abdomen. For example, circumferential liposuction of the thigh can theoretically cause prolonged postoperative edema by precipitating a vicious cycle and temporarily obliterating a significant portion of lymphatic drainage from the lower limb. This liposuction-induced edema produces a mild compartment syndrome with local hematic capillary ischemia, decreasing delivery of oxygenated blood, augmenting anaerobic metabolism, and increasing capillary permeability. This increased capillary permeability produces still more edema.

How well did you know this?
1
Not at all
2
3
4
5
Perfectly
68
Q

68- Which of the following sutures absorbs most rapidly?

A. Chromic gut
B. Polyglycolic acid
C. Polyglactin 91
D. Polydioxanone
E. Plain gut

A

Correct choice: E. Plain gut

Explanation: Of the absorbable sutures listed, plain gut absorbs most rapidly (absorption complete in 60-70 days). Chromic gut completely absorbs in ~80 days. Polyglactin 910 (Vicryl) and polyglycolic acid (Dexon) are both absorbed in about 90 days. Polydioxanone (PDS) is an absorbable monofilament which takes approximately 180 days to completely absorb.

How well did you know this?
1
Not at all
2
3
4
5
Perfectly
69
Q

69- Which of the following tattoo pigments is most commonly associated with allergic reactions, both eczematous and granulomatous?

A. Mercuric sulfide
B. Titanium dioxide
C. Cadmium sulfide
D. Ferric oxide
E. Iron oxide

A

Correct choice: A. Mercuric sulfide

Explanation: Allergic reactions have been reported with several different types of tattoo pigment. The most commonly associated tattoo pigment, however, is mercuric sulfide. Tattoos with mercuric sulfide appear red in color. Titanium dioxide (white tattoo color), Cadmium sulfide (yellow tattoo color), Ferric oxide (brown tattoo color), and Iron oxide (black tattoo color) are less commonly associated with allergic reactions. Of note, Cadmium sulfide is the tattoo pigment most commonly associated with phototoxic reactions.

How well did you know this?
1
Not at all
2
3
4
5
Perfectly
70
Q

A- 60-year-old female undergoes a surgical excision of a lipoma with complex linear closure on the left forearm. Approximately 24 hours later, she shows up in your dermatology clinic concerned about the following skin lesions present in the picture. You instruct the patient to avoid adhesives. Which is the next best step in this patient’s management?

A. Apply topical triple antibiotic ointment
B. Perform patch testing
C. Treat with a high-potency topical steroid
D. Treat with a burst of oral corticosteroids

E. Treat with non-steroidal anti-inflammatory agents

A

Correct choice: C. Treat with a high-potency topical steroid

Explanation: Allergic contact dermatitis to triple-antibiotic ointment or tape is a common complication or pitfall after surgery. The initial appropriate steps are to withdraw the likely source of allergen and treat with high-potency topical steroids. In the future, patch testing may be an appropriate investigation to pursue. Administration of non-steroidal anti-inflammatories is not necessary and is not the best single answer choice.

How well did you know this?
1
Not at all
2
3
4
5
Perfectly
71
Q

71- Which of the following is the most common genetic alteration seen in mucosal melanomas?

A. GNAQ
B. Cyclin Dependant Kinase 4/6
C. BRAF
D. KIT
E. MDM2

A

Correct choice: D. KIT

Explanation: Mucosal melanomas tend to have an activating mutation in KIT.
GNAQ mutations are seen in both uveal melanomas and blue nevi. Cyclin dependant kinase 4/6 binds cyclin D and together phosphorylate Retinoblastoma. MDM2 targets p53 for ubiquitination.

How well did you know this?
1
Not at all
2
3
4
5
Perfectly
72
Q

72- Which of the following anatomic structures is most likely to be severed during this repair?

A. Mental nerve
B. Angular artery
C. Buccal nerve
D. Inferior labial artery
E. Marginal mandibular nerve

A

Correct choice: D. Inferior labial artery

Explanation: The photograph demonstrates a wedge resection which is used to repair defects of the lower lip. During this repair, the inferior labial artery is likely to be severed since it courses superficially between the mucosa and the underlying muscle.
The angular artery, mental nerve, buccal nerve, and marginal mandibular nerve are unlikely to be severed during a wedge resection of the lower lip.

How well did you know this?
1
Not at all
2
3
4
5
Perfectly
73
Q
  1. A- patient of skin type III presents for ablative CO2 resurfacing. Which of the following is true?

A. The patient will have reduced risk of hyperpigmentation if they begin hydroquinione 2 weeks prior to the procedure
B. If the patient develops post-inflammatory hyperpigmentation, then hydroquinone is effective in treatment
C. Antivirals should not be given prior to procedure as they have not demonstrated any benefit in reduction of herpes simplex reactivation
D. The patient should have completed isotretinoin at least 3 months prior to CO2 laser resurfacing
E. CO2 lasers are ineffective in resurfacing

A

Correct choice: B. If the patient develops post-inflammatory hyperpigmentation, then hydroquinone is effective in treatment

Explanation: Hydroquinone does reduce post-inflammatory hyperpigmentation if it does develop after laser resurfacing. The 1999 paper by West and Alster demonstrated that pretreatment with hydroquinone did not reduce the risk of hyperpigmentation after CO2 laser resurfacing. Antivirals

have been shown, when prophylactically given, to reduce HSV outbreaks before medium depth peels and resurfacing in those prone to outbreaks. Isotertinoin should be completed 6-12 months prior due to atrophy of pilosebaceous units, thus increasing scarring risk after ablative CO2 laser resurfacing.

How well did you know this?
1
Not at all
2
3
4
5
Perfectly
74
Q

74-What is the treatment of choice for dermatofibrosarcoma protuberans?

A. wide local excision
B. cryotherapy
C. imatinib
D. Mohs surgery
E. observation

A

Correct choice: D. Mohs surgery

Explanation: Mohs surgery. Mohs surgery is the treatment of choice for DFSP. The local recurrence rate is 1% with Mohs surgery (vs. 15-50% with wide local excision). Imatinib is approved for unresectable or metastatic disease and targets the COL1A1-PDGFB fusion protein.

How well did you know this?
1
Not at all
2
3
4
5
Perfectly
75
Q
  1. A- patient sees you in your office for a cosmetic consultation for submental fullness. When discussing indications and risks of injecting deoxycholic acid (Kybella) you inform her that there is a risk of marginal mandibular nerve injury. A marginal mandibular nerve injury would manifest with which of the following:

A. Difficulty swallowing on the affected side
B. Frequent salivation on the affected side
C. Inability to chew properly on the affected side
D. Inability to elevate and retract the lip on the affected side
E. Inability to depress and retract the lip on the affected side

A

Correct choice: E. Inability to depress and retract the lip on the affected side

Explanation: The marginal mandibular nerve innervates the depressor anguli oris, depressor labii inferioris, mentalis, orbicularis oris, risorius, and platysma muscles. Depression and retraction of the lip is controlled by depressor anguli oris, depressor labii inferioris, and mentalis muscles. An injury to the marginal mandibular nerve would thus cause an inability to smile, and interfere with depression and retraction of the lip on the affected side. The mouth would seem normal at rest, but crooked when attempting to smile.
1 – A marginal mandibular nerve injury does not present with difficulty swallowing. 2 – A marginal mandibular nerve injury does not present with frequent salivation on the injured side. 3 – The buccal nerve innervates the buccinators which help to prevent accumulation of food between teeth and buccal mucosa with chewing. 4 – The buccal nerve innervates the zygomaticus minor, levator labii superioris, and risorius which elevate and retract the lip.

How well did you know this?
1
Not at all
2
3
4
5
Perfectly
76
Q

76- Within which anatomic plane does the artery that was encountered during this repair lie?

A. Submucosal
B. Dermal
C. Intramuscular
D. Submuscular
E. There is no major artery in this surgical field

A

Correct choice: A. Submucosal

Explanation: The inferior labial artery is derived from the facial artery or superior labial artery, and runs through the submucosal tissue horizontally.
The inferior labial artery does not typically lie in the other listed anatomic planes.

How well did you know this?
1
Not at all
2
3
4
5
Perfectly
77
Q

77- Regarding the use of chlorhexidine, which of the following is true?

A. Chlorhexidine rinses (0.12%) aid in prevention of drug-associated gingival enlargment
B. Chlorhexidine, when applied to the eyes, will reduce keratitis
C. Chlorhexidine reduces risk of deafness when applied to the ear canal prior to surgery
D. Chlorhexidine leads to reduction of suture strength when applied to wounds
E. The efficacy of chlorhexidine is reduced when combined with isopropyl alcohol

A

Correct choice: A. Chlorhexidine rinses (0.12%) aid in prevention of drug-associated gingival enlargment

Explanation: Chlorhexidine rinses help prevent drug-associated gingival enlargement. Chlorhexidine is irritating to eyes and middle ear. It has no effect on suture strength. ChloraPrep is a combination of chlorhexidine and isopropyl alcohol.
Reference: Reference: Dermatology. Bolognia, Jorizzo, Schaffer. 3rd ed. Ch 72.

How well did you know this?
1
Not at all
2
3
4
5
Perfectly
78
Q
  1. A- male patient with androgenetic alopecia is seen for hair transplant consultation. He tried topical minoxidil and oral finasteride. You want to set realistic expectations for this patient. How should you counsel this patient?

A. Stop minoxidil and finasteride, he will no longer need it after hair transplant
B. Androgenetic alopecia progresses until age 50
C. Medical therapy is used in conjunction with hair transplant to maximize hair density
D. Additional hair transplants will not be needed in the future
E. The hairs grown or maintained by use of minoxidil and finasteride will stay in place once these medications are stopped

A

Correct choice: C. Medical therapy is used in conjunction with hair transplant to maximize hair density

Explanation: Medical therapy such as minoxidil and finasteride are often used in conjunction with hair transplantation to maximize density and minimize ongoing hair loss. Patients should be counseled that androgenetic alopecia (AGA) is an ongoing process. The progressive nature of AGA means that additional hair transplants may be required in 5-10 years. The physician should emphasize before the procedure how ongoing hair loss will affect the density and cosmetic appearance of the transplant. If medications are discontinued, the patient can expect loss of hairs grown or maintained by their use.

How well did you know this?
1
Not at all
2
3
4
5
Perfectly
79
Q

79- The patient had an extensive superficial basal cell carcinoma on the forehead treated with topical imiquimod and returned 4-weeks later with this reaction. Which cytokine is not activated by this medication?

A. tumor necrosis factor (TNF)-α
B. interferon (IFN)-α
C. interferon (IFN)-γ
D. interleukin (IL)-12
E. interleukin (IL)-4

A

Correct choice: E. interleukin (IL)-4

Explanation: Imiquimod is a topical immunomodulator that is FDA-approved for the treatment of genital warts, actinic keratosis and superficial basal cell carcinomas. It induces toll like receptor (TLR) 7 and the production of Th-1 cytokines including tumor necrosis factor (TNF)-α , interferon (IFN)-γ , interferon (IFN)-α , and interleukin (IL)-12, leading to stimulation of a cell-mediated immune response. IL-4, IL-5, IL-6 and IL-13 are Th-2 cytokines.

How well did you know this?
1
Not at all
2
3
4
5
Perfectly
80
Q

80- What is the name of the instrument “1”?

A. Bishop-Harmon forceps
B. Adson forceps
C. Kelly forceps
D. Castroviejo forceps
E. Jewelers forceps

A

Correct choice: A. Bishop-Harmon forceps

Explanation: Bishop-Harmon forceps
Reference: Reference: Dermatology. Bolognia, Jorizzo, Schaffer. 3rd ed. Ch 144.

How well did you know this?
1
Not at all
2
3
4
5
Perfectly
81
Q
  1. A- patient recently underwent parotid surgery and now reports unilateral flushing and sweating around mealtime. The nerve injured in this syndrome is a branch of which nerve?

A. Facial nerve
B. Maxillary nerve
C. Mandibular nerve
D. Cervical nerve
E. Frontal nerve

A

Correct choice: C. Mandibular nerve

Explanation: This patient has Frey syndrome or auriculotemporal nerve syndrome. This is characterized by facial flushing, sweating, or both localized to the distribution of the auriculotemporal nerve (a branch of the mandibular nerve) that occurs in response to gustatory stimuli. In adults, the syndrome usually results from surgical injury or trauma to the parotid gland.

How well did you know this?
1
Not at all
2
3
4
5
Perfectly
82
Q

82-A 75-year-old man is preparing for excision of the shown lesion on his trunk. His preoperative evaluation reveals that he has a pacemaker and also is known to have mitral valve prolapse without

valvular regurgitation. According to the American Heart Association, which of the following antibiotic dosing regimens is indicated prior to his surgery?

A. Amoxicillin, 2 g, 1 hour before the procedure
B. Amoxicillin, 1 g, 1 hour after the procedure
C. Azithromycin, 1 g, 1 hour before the procedure
D. Clindamycin, 500 mg, 1 hour after the procedure
E. No antibiotics

A

Correct choice: E. No antibiotics

Explanation: This patient does not require preoperative antibiotics.
The presence of implantable defibrillators and pacemakers are not indications for antibiotic use. Patients with mitral valve prolapse without valvular insufficiency or regurgitation noted on ultrasound are at no higher risk for bacterial endocarditis than the general population and, therefore, do not require preoperative antibiotics.
According to the AHA, those patients who are at high risk for bacterial endocarditis and require preoperative antibiotics include those with prosthetic cardiac valves, previous bacterial endocarditis, complex cyanotic congenital heart disease, and surgically constructed systemic-pulmonary shunts. Moderate risk patients include those with mitral valve prolapse with regurgitation, acquired valvular dysfunction, and hypertrophic cardiomyopathy

How well did you know this?
1
Not at all
2
3
4
5
Perfectly
83
Q

83-Which of the following suture materials induces the least inflammation?

A. Surgical gut
B. Polyglycolic acid (Dexon)
C. Polyglycan 910 (Vicryl)
D. Polypropylene (Prolene)

E. Silk

A

Correct choice: D. Polypropylene (Prolene)

Explanation: Prolene is a nonabsorbable suture material that evokes only minimal inflammation. Vicryl, Dexon, silk and surgical gut are associated with more inflammation than prolene.

How well did you know this?
1
Not at all
2
3
4
5
Perfectly
84
Q

A- Z-plasty represents which type of surgical reconstruction?

A. Complex linear layered closure
B. Advancement flap
C. Rotation flap
D. Transposition flap
E. Interpolation flap

A

Correct choice: D. Transposition flap

Explanation: A Z-plasty is a type of transposition flap that is useful for scars crossing relaxed tensions lines or releasing contractures (redistributes tension over wound).

A Z-plasty involves the movement of tissue via transpositioning across intervening islands of unaffected tissue. Therefore, the remaining answer choices are not correct.

How well did you know this?
1
Not at all
2
3
4
5
Perfectly
85
Q

85- Endocarditis prophylaxis is recommended for which of the following?

A. Rheumatic heart disease
B. Atrial septal defects
C. Hypertrophic cardiomyopathy
D. Mitral valve prolapse without regurgitation
E. Previous bacterial endocarditis

A

Correct choice: E. Previous bacterial endocarditis

Explanation: Preoperative antibiotics are recommended for endocarditits prophylaxis in select high risk patients that undergo high risk procedures. High risk procedures are procedures that involve manipulation of gingival tissue or the periapical region of teeth or perforation of the oral mucosa. High risk patients are, according to the guidelines, those with the following: Prior infective endocarditis, Prosthetic cardiac valves, Unrepaired cyanotic congenital heart defects, including palliative shunts and conduits, congenital heart defects completely repaired with prosthetic material or a device, whether placed by surgery or by catheter intervention, during the first 6 months after the procedure, repaired congenital defects with residual defects at the site or adjacent to the site of a prosthetic patch or prosthetic device, cardiac transplants and development of cardiac valvulopathy. Patient groups that may have received routine antibiotic prophylaxis in the past but are no longer candidates for it include those with mitral and aortic valve disease, rheumatic heart disease, or structural disorders like ventricular or atrial septal defects or hypertrophic cardiomyopathy, according to the AHA statement. The revised guidelines were developed with the participation of and have been endorsed by the American Dental Association, the Infectious Diseases Society of America, and the American Academy of Pediatrics. Classically 1 gram of Dicloxacillin or cephalexin is given 1 hour preoperatively and an additional 500 mg is given 6 hours post op. Clindamycin can be givenin those patients who are penicillin allergic.

Patient groups that may have received routine antibiotic prophylaxis in the past but are no longer candidates for it include those with mitral and aortic valve disease, rheumatic heart disease, or structural disorders like ventricular or atrial septal defects or hypertrophic cardiomyopathy, according to the AHA statement.

How well did you know this?
1
Not at all
2
3
4
5
Perfectly
86
Q

86- What is the optimal time to take down this flap?

A. 3 weeks
B. 1 week
C. 2 weeks
D. 4 weeks
E. 3 days

A

correct choice: A. 3 weeks

Explanation: This is an interpolation flap and the optimal time for take down is 3 weeks after the flap is done.

How well did you know this?
1
Not at all
2
3
4
5
Perfectly
87
Q

87- What is the name of the pictured instrument?

A. Westcott scissors
B. Gradle scissors
C. Metzenbaum scissors
D. Adson scissors
E. Iris scissors

A

correct choice: B. Gradle scissors

Explanation: The pictured instrument is a gradle scissors. The tip has a curvature which approximates a cradle, which rhymes with gradle.

How well did you know this?
1
Not at all
2
3
4
5
Perfectly
88
Q
  1. A- 30-year-old female presents to your cosmetic dermatology clinic hoping to improve severe atrophic acne scarring on both cheeks which has been present since adolescence. Which of the following is the preferred resurfacing modality and gold standard to improve this patient’s scarring?

A. Pulsed dye laser
B. Motorized dermabrasion
C. Intense pulsed light
D. Carbon dioxide laser
E. 35% trichloroacetic acid chemical peel

A

Correct choice: B. Motorized dermabrasion

Explanation: Moderate to severe acne scarring is the most notable condition for which motorized dermabrasion is the preferred resurfacing modality. Laser resurfacing has yielded variable results and chemical peeling is generally disappointing. Dermabrasion selectively planes the affected skin. Carbon dioxide laser ablation and trichloroacetic acid chemical peels may efface the edges of atrophic scars but are more difficult with less predictable results compared to dermabrasion. Conventional motorized dermabrasion remains the gold standard in resurfacing procedures for atrophic or boxcar acne scars.

1 – Pulsed dye laser is not an effective resurfacing treatment option for moderate to severe acne scarring. 3 – Intense pulsed light is not an effective resurfacing treatment option for moderate to severe acne scarring. 4 – Effacing the edges of atrophic scars can be accomplished with the carbon dioxide laser but may be more difficult and with less predictable results compared to dermabrasion. 5 – Chemical peels are generally disappointing for moderate to severe acne scarring. Effacing the edges of atrophic scars can be accomplished with a 35% trichloroacetic acid peel but is more difficult and has less predictable results compared with dermabrasion.

How well did you know this?
1
Not at all
2
3
4
5
Perfectly
89
Q

89- What artery is often encountered during this repair?

A. Angular artery
B. Inferior labial artery
C. Supratrochlear artery
D. Supraorbital artery
E. Anterior ethmoidal artery

A

Correct choice: A. Angular artery

Explanation: The angular artery courses up the nasolabial fold and is deep to the nasal alar groove and is often encountered with this V to Y repair. The other arteries are not located in this area. The anterior ethmoidal nerve supplies sensation to the columella and nasal tip.

How well did you know this?
1
Not at all
2
3
4
5
Perfectly
90
Q

90- The operational wavelength of the most common laser used to treat the disorder depicted in this image is:

A. 1064 nm
B. 308 nm
C. 595 nm
D. 755 nm
E. 532 nm

A

Correct choice: B. 308 nm

Explanation: The operational wavelength of the 308 nm excimer laser and lamp is close to that of NB-UVB. The therapeutic benefit of the excimer laser for vitiligo has been investigated in multiple studies, and, overall, 20–50% of lesions achieve ≥75% repigmentation; the excimer lamp appears to have similar efficacy. Localized patches of vitiligo are treated one to three times weekly for an

average of 24 to 48 sessions, with the repigmentation rate depending on the total number of sessions, not their frequency. Erythema and (rarely) blistering represent potential side effects.

How well did you know this?
1
Not at all
2
3
4
5
Perfectly
91
Q

91- Which filler is least likely to cause nodules in the area indicated?

A. Poly-L-lactic acid
B. Hyaluronic acid
C. Polymethyl methacrylate
D. Silicone
E. Calcium hydroxyapatite

A

Correct choice: B. Hyaluronic acid

Explanation: Hyaluronic acid. Poly-L-lactic acid (Sculptra), polymethyl methacrylate (Artefill/ Artecoll), silicone and calcium hydroxyapatite (Radiesse) are all much more likely to cause nodules in the perioral area than hyaluronic acid fillers.

How well did you know this?
1
Not at all
2
3
4
5
Perfectly
92
Q

92- What is the purpose of this design?

A. To increase eversion of the wound
B. To increase the length of the wound
C. To avoid important anatomic structures
D. To provide better hemostasis
E. To increase wound tension

A

Correct choice: C. To avoid important anatomic structures

Explanation: This is a diagram of a M-plasty, which is used to shorten the length of a scar in order to avoid important anatomic structures. The other choices are untrue.

How well did you know this?
1
Not at all
2
3
4
5
Perfectly
93
Q

93- What is the etiology of this complication?

A. Poor hemostasis
B. Poor blood supply
C. Pulling sutures too tight
D. Not enough undermining
E. Too large of a flap for the defect

A

Correct choice: D. Not enough undermining

Explanation: This is a photograph of trap door deformity which occurs when not enough undermining is performed.

How well did you know this?
1
Not at all
2
3
4
5
Perfectly
94
Q

94.A- 60-year-old man presents with a tendency to flush combined with a background of persistent facial erythema. On exam, you notice a background of telangiectasias. He has maximized treatment with topical and oral medications and is considering laser treatment for his prominent blood vessels. Which is the laser of choice for this patient’s condition?

A. Pulsed dye laser
B. KTP laser
C. Intense pulsed light
D. Diode laser
E. Nd:YAG laser

A

Correct choice: A. Pulsed dye laser

Explanation: The pulsed dye laser (PDL) is considered to be the laser of choice for the treatment of many cutaneous vascular lesions, especially facial telangiectasias, port-wine stains, superficial hemangiomas, poikiloderma of Civatte, and erythematotelangiectatic rosacea.

B - KTP laser: KTP laser can be used to effectively treat some vascular lesions (e.g. telangiectasias) in a purpura-free manner. While it can be used to treat vascualr lesions including telangiectasias, pulsed dye laser is the laser of choice for treatment of cutaneous vascular lesions such as telangiectasias.

C - Intense pulsed light: IPL is most commonly used for patients with phototypes IIII with chronic photodamage or for hair removal. For treatment of patients with more darkly pigmented skin, longer wavelength filters or lower fluences plus longer pulse widths can be used. Filters can also be used for other specific indications, e.g. treatment of vascular lesions. Although it can be used to treat vascular lesions, PDL is considered to be the laser of choice for treatment of facial telangiectasias. D - Diode laser: Diode lasers can be used to treat leg venulectasias and telangiectasias (“spider veins”), blue reticualr veins, and venous malformations. However, a pulsed dye laser is considered to be the laser of choice for treatment of facial telangiectasias.

E - Nd:YAG laser: The deeply penetrating Nd:YAG 1064 nm laser has significantly less hemoglobin absorption than the wavelengths of the PDL or KTP lasers. This laser can be used to treat deeper vascular lesions and spider veins up to 3 mm in size. Because of its poor coefficient of absorption by hemoglobin, much higher fluences are required to produce transmural heating of blood vessels. Nd:YAG lasers should be used with extra caution when treating vascular lesions.

How well did you know this?
1
Not at all
2
3
4
5
Perfectly
95
Q

95- What laser wavelength best targets these lesions?

A. 1550nm
B. 10600nm
C. 532nm
D. 755nm
E. 2940nm

A

Correct choice: C. 532nm

Explanation: These are telangiectasias which are most effectively treated with the long pulsed 532nm KTP laser. The 1550nm, the CO2 10600nm and the erbium 2940nm lasers target water and do not target the hemoglobin. The alexandrite 755nm targets melanin not hemoglobin

How well did you know this?
1
Not at all
2
3
4
5
Perfectly
96
Q

96- A 55 y/o male has multiple skin tags on the upper eyelids and presents to discuss cosmetic removal. Which of the following instruments would be the most appropriate tool to remove these lesions?

A. Large Metzenbaum scissors

B. Spencer scissors
C. Baby Metzenbaum scissors
D. Obrien scissors
E. Gradle scissors

A

Correct choice: E. Gradle scissors

Explanation: Gradle scissors have delicate sharp tips, making them ideal for skin tag removal. Obrien scissors and Spencer scissors are suture scissors. Obrien scissors have angled tips, whereas Spencer have a characteristic round suture hook. Baby and large Metzenbaum scissors have blunt tips and are used for undermining tissue.

How well did you know this?
1
Not at all
2
3
4
5
Perfectly
97
Q

97- This patient presents for Mohs surgery. What is the deepest layer that you can undermine in this area?

A. Superficial muscular aponeurotic system (SMAS)
B. Muscle
C. Subcutaneous fat
D. Dermis
E. Epidermis

A

Correct choice: C. Subcutaneous fat

Explanation: To avoid injury to the temporal branch of the facial nerve in this area, undermining should occur in the subcutaneous fat layer. Undermining deeper than the subcutaneous fat layer (SMAS or muscle) could risk injury to the temporal branch of the facial nerve.

How well did you know this?
1
Not at all
2
3
4
5
Perfectly
98
Q

98- A patient develops steroid atrophy 1 cm in size after intralesional steroid for acne cyst on the right cheek. What is a reasonable, least costly, and minimally invasive first step?

A. Microneedling with platelet rich plasma to the area involved
B. Excision of area with intermediate layered linear repair
C. Serial normal saline injections to the area
D. Fat transfer from the abdomen to the area
E. Hyaluronic filler injection to the area

A

Correct choice: C. Serial normal saline injections to the area

Explanation: Saline injections in serial fashion are the least invasive, least costly option to offer the patient. Injections are performed every 1-2 weeks for at least 2 months before evaluating for success or failure. The remaining choices are incorrect as the first, least invasive option.

How well did you know this?
1
Not at all
2
3
4
5
Perfectly
99
Q

99- A 75 y/o male presents with a lesion - biopsy is shown below. Which of the following antiseptics was likely used in the excision of this lesion?

A. Isopropyl alcohol
B. Hexachlorophene
C. Povidone-iodine
D. Normal Saline
E. Chlorohexidine

A

Correct choice: C. Povidone-iodine

Explanation: The image shows a pathologic photo of sebaceous carcinoma. The most common location of sebaceous carcinoma is the upper eyelid. Given this location, povidone-iodine is the best answer. Of note, it must be allowed to completely dry before starting surgery. Chlorohexidine’s exposure to the eyes can cause keratitis and therefore should be avoided in these areas.
Isopropyl alcohol has not demonstrated toxic effects on the eyes and can be used safely, but is typically too short acting for surgery. Hexachlorophene causes neurotoxicity and is teratogenic so is rarely used today. Normal Saline has not demonstrated toxic effects on the eyes and can be used safely, but does not have antiseptic properties.

How well did you know this?
1
Not at all
2
3
4
5
Perfectly
100
Q

100- What filler is contraindicated in this area?

A. Poly-L-lactic acid
B. Calcium hydroxyapatite
C. Silicone
D. Bovine collagen
E. Hyaluronic acid

A

Correct choice: D. Bovine collagen

Explanation: Bovine collagen is contraindicated in the glabella due to necrosis, however, extreme caution must be used when injecting any filler in the glabellar region as it is a watershed area.
Injection necrosis is a rare but clinically important potential complication caused by interruption of the vascular supply to the area by compression, injury, and/or obstruction of the vessel(s). The glabella is a particular danger zone for injection necrosis regardless of the type of filler used, however bovine collagen poses a greater risk of developing this side effect.

How well did you know this?
1
Not at all
2
3
4
5
Perfectly
101
Q

101- You perform a melanoma excision on a patient’s back and leave it open to wait for pathology results. On day 5, results come back with clear margins. You call the patient back and perform an intermediate layered closure on day 5. How would you bill this repair on day 5?

A. 12032 with 79 modifier
B. 12032 with 58 modifier
C. 12032 with 25 modifier
D. 12032 with 59 modifier
E. 12032

A

Correct choice: B. 12032 with 58 modifier

Explanation: This question describes a malignant excision, which has a 10 day global period. Therefore, the repair that was performed on day 5 in this scenario falls into this global period. Given this, the repair code 12032 will require a modifier. The correct answer is the 58 modifier, which describes a related/staged procedure by the same physician within a global period.

The 25 modifier designates a separate E/M on the same day as a procedure.
The 59 modifier is a clinically distinct procedure performed on the same day by the same physician. The 79 modifier designates an unrelated procedure by the same physician in a global period.

How well did you know this?
1
Not at all
2
3
4
5
Perfectly
102
Q

102- What is the optimal angle for the apices of this surgical procedure?

A. 45
B. 30
C. 60
D. 90
E. 15

A

Correct choice: D. 30

Explanation: This is an ellipse, and the optimal angle between the apices is 30 degrees. In rhombic transposition flaps, the rhombus consists of two 60-degree angles and two 120-degree angles.

How well did you know this?
1
Not at all
2
3
4
5
Perfectly
103
Q

103- A patient presents to you for a new rash on the trunk. You had previously performed Mohs Surgery 60 days ago for a BCC on the nose, repaired with a bilobed transposition flap. How would you code this visit?

A. 99213 with 59 modifier
B. 99213 with 25 modifier
C. 99213 with 24 modifier
D. 99213 with 79 modifier
E. 99213
c

A

Correct choice: C. 99213 with 24 modifier

Explanation: Adjacent tissue transfer/Flap reconstruction after Mohs surgery has a 90 global period. This visit is an unrelated E/M during a global period, and would therefore require a modifier 24.
Modifier 25 designates a procedure on the same day/visit as an E/M.
Modifier 59 designates 2 separate and distinct procedures were performed on the same day. Modifier 79 is a separate procedure performed by the same physician during a global period.

How well did you know this?
1
Not at all
2
3
4
5
Perfectly
104
Q

104- In solid organ transplant recipients, all of the following risk factors increase susceptibility to the development of post-transplant skin cancer EXCEPT:

A. Thoracic organ transplantation
B. Fair skin (Fitzpatrick types I-III)
C. Immunosuppression with mTor inhibitor
D. Increased age at time of transplant
E. Male sex

A

Correct choice: C. Immunosuppression with mTor inhibitor

Explanation: All organ transplant recipients are at increased risk for the development of cutaneous malignancies. However, the above-mentioned factors, with the exception of choice E, place these individuals at further risk. Sirolimus (mammalian target of rapamycin inhibitor) has shown evidence of decreasing the risk of post-transplantation skin cancer incidence. Post-transplant skin cancer is common, with elevated risk imparted by increased age, white race, male sex, and thoracic organ transplantation.

How well did you know this?
1
Not at all
2
3
4
5
Perfectly
105
Q

105- A 69 y/o patient presented to her ophthalmologist with burning pain, tearing, and blurred vision in one of her eyes. She reported that she had a cosmetic procedure performed at a local med spa the day prior. What procedure did she likely have performed?

A. All procedures could have caused this injury
B. Q-switched Alexandrite for facial lentigines
C. KTP for nasal telangiectasias
D. PDL for facial redness
E. CO2 laser resurfacing for facial wrinkles

A

Correct choice: E. CO2 laser resurfacing for facial wrinkles

Explanation: This stem describes a woman with a corneal ulceration. This typically presents with burning pain at the site of exposure followed by tearing and blurred vision.

CO2 lasers have wavelengths of 10,600 nm which targets the chromophore of water. Lasers > 800 nm can damage the cornea, which is a water containing structure. Therefore, exposure of CO2 to the eye can cause corneal injuries. Nd:YAG (1320 nm), diode (1440 nm), Er:glass (1550 nm), Er:YAG (2940 nm) can also cause corneal damage.
Lasers < 800 nm damage the retina, iris, uvea, and choroid (pigment-containing structures). PDL has target wavelength of 585-595 nm
Q-Alex has target wavelength of 755 nm KTP has target wavelength of 532 nm
These wavelengths are therefore too short to damage the cornea.

How well did you know this?
1
Not at all
2
3
4
5
Perfectly
106
Q

106- What nerve innervates this structure?

A. Infratrochlear
B. Anterior ethmoidal
C. Infraorbital
D. Zygomaticotemporal
E. Buccal

A

Correct choice: B. Anterior ethmoidal

Explanation: The external nasal nerve innervates the columella and nasal tip. It is a branch of the anterior ethmoidal nerve, which provides branches of V1. The infratrochlear is also a branch of V1 but it supplies the nasal root and medial canthus. The infraorbital and zygomaticotemporal are branches of V2. The buccal is a branch of V3.
Most of the sensory innervation of the nasal cavity is derived from two sources: the sphenopalatine ganglion and the anterior ethmoidal nerve. The sphenopalatine ganglion (pterygopalatine, nasal, or Meckel’s ganglion) is a parasympathetic ganglion that is located in the pterygopalatine fossa, posterior to the middle turbinate. Its sensory root is derived from sphenopalatine branches of the maxillary nerve, cranial nerve (CN) V2. As they pass through the sphenopalatine ganglion, these sensory branches form the greater and lesser palatine nerves, which provide sensory innervation to the nasal cavity as well as the roof of the mouth, soft palate, and tonsils. The anterior ethmoidal nerve is one of the sensory branches of the ciliary ganglion, which is located within the orbital cavity and inaccessible to nerve blocks. It provides sensory innervation to the anterior portion of the nasal cavity.

How well did you know this?
1
Not at all
2
3
4
5
Perfectly
107
Q

107- Which of the following is a potential side effect of ambulatory tumescent liposuction?

A. Muscle atrophy
B. Breast enlargement

C. Decreased appetite
D. Abdominal perforation
E. Pulmonary embolus

A

►B

Breast enlargement is a relatively common and unexpected side effect of tumescent liposuction. Since the majority of these patients report increased breast size in the absence of weight gain, some authors postulate shifting hormone ratios as the etiology of this paradoxical breast augmentation.Abdominal perforation, respiratory failure and pulmonary embolus are complications that are seen almost exclusively in liposuction patients that receive general anesthesia or intravenous sedation.

How well did you know this?
1
Not at all
2
3
4
5
Perfectly
108
Q

108 -Which is an advantage of a split-thickness skin graft over a full-thickness skin graft?
A. Better color match
B. Better texture match
C. Less contraction
D. Decreased infection risk
E. Improved survival

A

►E

The major advantage of a split-thickness skin graft over a full-thickness graft is the increased survivability. Full-thickness skin grafts have better color and texture match and less contraction. There is no significant difference in infection risk.

How well did you know this?
1
Not at all
2
3
4
5
Perfectly
109
Q

109 -Which of the following sutures is the first to be absorbed?

A. Catgut
B. Polyglycolic acid
C. Polyglactin 910
D. Polydioxanone
E. Polypropylene

A

►A

Catgut has a variable rate of absorption but typically lasts about 7-14 days. Thus, it would be the first suture to be absorbed. Polyglactin 910 (Vicryl) and polyglycolic acid (Dexon) are both absorbed in about 90 days. Polydioxanone (PDS) is an absorbable monofilament which lasts approximately 180 days. Polypropylene (Prolene) is not an absorbable suture.

How well did you know this?
1
Not at all
2
3
4
5
Perfectly
110
Q

110- Which of the following injectable local anesthetics is the longest lasting?

A. Lidocaine
B. Bupivacaine
C. Meppivacaine
D. Procaine
E. Diphenhydramine

A

►B

Bupivacaine is the longest lasting injectable local anesthetic-120-240 minutes without epinephrine, 180-420 minutes with epinephrine. Lidocaine lasts 30-60 minutes without epinephrine, 120-360 with epinephrine. Mepivacaine last 45-90 minutes without epinephrine, 120-360 with epinephrine. Procaine lasts 15-60 minutes. Diphenhydramine lasts 30 minutes.

How well did you know this?
1
Not at all
2
3
4
5
Perfectly
111
Q

111 -Which of the following demonstrates the highest risk of metastasis?

A. SCC on the ear
B. SCC within a scar
C. SCC on the lip
D. SCC on the scalp
E. SCC on the nose

A

►B

Location of tumor is an important risk factor for metastasis of squamous cell carcinoma. Compared with a 10% likelihood of metastasis for tumors located on the ear or the lip, an SCC developing in the scar, however, has been estimated to metastasize at a rate as high as 30-40%. .

How well did you know this?
1
Not at all
2
3
4
5
Perfectly
112
Q

112- The post-operative complication shown in the photograph is most commonly seen with which type of reconstruction?

A. Rhombic flap
B. Nasolabial flap
C. Rotation flap
D. Full-thickness skin graft
E. Secondary intention

A

►B

Trap door deformity is believed to result from insufficient undermining. This surgical complication is most often associated with the nasolabial transposition flap. Intralesional corticosteroids may be beneficial in improving the cosmetic outcome.

How well did you know this?
1
Not at all
2
3
4
5
Perfectly
113
Q

113- Which is the following is true regarding cellulite?

A. It is caused by lipoatrophy of fat.
B. Adipocytes in the gluteofemoral region are more responsive to lipolysis.
C. Norepinephrine is the only hormone that affects lipolysis.
D. It occurs in up to 50% of postpubertal females.
E. Caucasian women tend to get cellulite more than Asian women.

A

►E

According to the JAAD CME article in March 2010, cellulite is caused by the herniation of subcutaneous fat, not lipoatrophy, within fibrous connective tissue. It is prevalent in almost all postpubertal females. It is most notable in the pelvic region, lower limbs, and abdomen. The adipocytes in the gluteofemoral region are larger and are influenced by female sex hormones. They

are metabolically more stable and resistant to lipolysis. The hormones that acutely affect lipolysis in adipocytes are catecholamines (epinephrine and norepinephrine) and insulin.

How well did you know this?
1
Not at all
2
3
4
5
Perfectly
114
Q

114- Which of the following lasers has the greatest depth of penetration in the skin?

A. Pulsed dye laser (585 nm)
B. Diode (800 nm)
C. Nd: YAG (1064 nm)
D. Erbium: YAG (2940 nm)
E. CO2 (10,600 nm)

A

►C

The Nd: YAG laser emits energy at 1064nm near the infrared range and penetrates the skin to the level of the deep dermal blood vessels. The target chromophore for this laser is melanin, thus its primary use is in treating lesions such as nevus of Ota and removing black tattoo pigment. Although both the erbium:YAG and CO2 lasers have longer wavelengths than the Nd: YAG, they are ablative lasers which only penetrate to the level of the stratum corneum and superficial epidermis, respectively.

How well did you know this?
1
Not at all
2
3
4
5
Perfectly
115
Q

115- The maximum dose of lidocaine with epinephrine when used in tumescent anesthesia is:

A. 3 mg/kg
B. 4.5 mg/kg
C. 7 mg/kg
D. 20 mg/kg
E. 50 mg/kg

A

►E

The maximum recommended dosage of lidocaine in adults is 4.5 mg/kg without epinephrine, 7.0 mg/kg with epinephrine, and 55 mg/kg in tumescent anesthesia for liposuction.

How well did you know this?
1
Not at all
2
3
4
5
Perfectly
116
Q

116 -Which of the following chemical peels does not need to be neutralized?

A. Salicylic acid
B. TCA 40%
C. Glycolic acid
D. Lactic acid
E. Phenol

A

►A

Salicylic acid is a very superficial chemical peel that is used for acne, milia, keratolysis. This peel localizes to the pores given its lipophilic nature. It is a self-neutralizing peel.

How well did you know this?
1
Not at all
2
3
4
5
Perfectly
117
Q

117- The use of imiquimod (Aldara) for the treatment of superficial basal cell carcinoma is advocated at which treatment regimen?

A. Five times a week for 6 weeks
B. Five times per week for 4 weeks
C. Three times a week for 6 weeks
D. Three times per week for 4 weeks
E. Three times a week for 10 weeks

A

►A

Imiquimod (Aldara) was FDA-approved in 2004 for the treatment of superficial basal cell carcinoma. The recommended treatment schedule is once daily, five times per week for a total of six weeks. Geisse J et al. Imiquimod 5% cream for the treatment of superficial basal cell carcinoma: results from two phase III, randomized, vehicle-controlled studies.

How well did you know this?
1
Not at all
2
3
4
5
Perfectly
118
Q

118 -Which of the following topical antibacterial agents may cause neutropenia?

A. Mupirocin
B. Silver sulfadiazine

C. Polymyxin
D. Bacitracin
E. Povodine-iodine

A

►B

Silver sulfadiazine is most commonly used to prevent infection in second and third-degree burn patients. Rare cases of leukopenia, neutropenia and kernicterus have been reported in patients using this topical antibacterial agent. Consequently, silver sulfadiazine should be used cautiously and avoided in infants, nursing mothers and pregnant women. Silver sulfadiazine also has the potential to cause a hypersensitivity reaction in patients with sulfa allergy.

119
Q

119- The surgical instrument shown in the photograph is a:

A. Needle holder
B. Hemostat
C. Gradle scissor
D. Towel clamp
E. Iris scissor

A

►B

A hemostat is shown in the photograph. This instrument is commonly used in cutaneous surgery for clamping off blood vessels.

120
Q

120- Which one of the following best estimates the percent strength of a wound one month after surgery with a primary closure?

A. 30%
B. 50%
C. 70%
D. 80%
E. 90%

A

►B

One month after surgery with a primary closure a wound is approximately at 50% of its ultimate strength (this is 35% of the original strength of the skin). Two weeks after surgery it is at 5% strength. Three weeks after surgery it is at 20%. It never reaches 100% of its original strength, its maximum strength is 90%.

121
Q

121 -The most appropriate laser to treat a port wine stain on an infant‟s cheek would have which of the following wavelengths?

A. 488 nm
B. 532 nm
C. 585 nm
D. 694 nm
E. 810 nm

A

►C

The 585nm pulsed dye laser (PDL) targets intravascular oxyhemoglobin and is considered the treatment of choice for most benign vascular lesions. The original PDL had a wavelength of 577 nm which was later modified to 585 nm to achieve deeper penetration yet still maintainvascular specificity.

122
Q

122 -A patient with a basal cell carcinoma on the lower eyelid has Mohs surgery. Once tumor-free margins were obtained, the surgeon repairs the wound with a full thickness skin graft. Which of the following statements is true about this repair method?

A. Graft should be at least 25% larger than the defect
B. Graft rarely survives in this location
C. Graft should be the same size as the defect
D. Healing by secondary intention would minimize the risk of ectropion
E. Graft should be at least 25% smaller than the defect

A

►A

To account for wound contraction and minimize the risk of ectropion, full thickness skin grafts on the lower eyelid should be sized such that the graft is at least 25% larger than the actual size of the wound.

123
Q

123- Jessner‟s solution contains all of the following except:

A. Salicylic acid
B. Resorcinol
C. Glycolic acid
D. Ethanol
E. Lactic acid

A

►C

Jessner‟s solution is a combination of resorcinol, salicylic acid, and lactic acid in ethanol that is used as a superficial peeling agent. The advantage of Jessner‟s solution is that timing is unnecessary and neutralization is not performed. The lactic acid is an alpha hydroxy acid.

124
Q

124 -All of the following statements are true regarding imiquimod (Aldara) except:

A. It is FDA-approved for the treatment of actinic keratoses
B. It is FDA-approved for the treatment of a 1.2 cm superficial basal cell on the scalp
C. It is FDA-approved for the treatment of a 1.9 cm superficial basal cell on the chest
D. It is FDA-approved for the treatment of a 1.5 cm superficial basal cell on the leg
E. It is FDA-approved for the treatment of a 2.0 cm superficial basal cell on the back

A

►B

Imiquimod (Aldara) is FDA-approved for the treatment of actinic keratoses and the treatment of superficial basal carcinoma. With respect to basal cell carcinoma, treatment is indicated for primary tumors that are 2.0 cm or less, and that are located on areas of the body excluding the face, scalp and anogenital region.

125
Q

125 -A physician using a carbon dioxide laser to treat verruca should be aware that all of the following complications can occur except:

A. Ocular damage
B. Transmission of viral disease
C. Purpura
D. Recurrence of lesion
E. Scarring

A

►C

The carbon dioxide laser uses a 10,600 nm wavelength to target water as a chromophore. Lasers that target water may damage the cornea. The carbon dioxide laser can be used to destroy epidermal and dermal lesions such as warts. HPV virus has been recovered in the laser plume after treatment with carbon dioxide laser. Bovine papilloma virus recovered in the plume was even found to transmit disease to calf skin. Scarring and recurrence have also been found to be potential complications.

126
Q

126 -The anatomic structure identified in the photograph is called the:

A. Soft triangle
B. Columella
C. Cupid‟s bow
D. Philtral ridge
E. Vermillion border

A

►D

The philtral ridges are located at the upper middle lip and form part of the aesthetically important cupid‟s bow.

127
Q

127- Which topical antiseptic works via denaturation of proteins?

A. Alcohols
B. Chlorhexidine
C. Iodine
D. Hexachlorophene
E. Triclosan

A

►A

Alcohols exert their antimicrobials effect via denaturation of proteins ie DNA, RNA, lipids, etc. Chlorhexidine, hexachlorophene, and triclosan disrupt the microbial cell membrane. Iodine and iodophores work via oxidation/substitution for elemental form of iodine, thereby precipitating proteins.

128
Q

128- While contemplating the repair of a large cheek defect after Mohs surgery, you begin to anesthetize an elderly lady who weighs 110 lbs. What is the maximum amount of 1% lidocaine with 2.5% epinephrine that this patient can receive?

A. 10 ml
B. 35 ml
C. 50 ml

D. 350 ml
E. 500 ml

A

►B

The maximum recommended dosage of 1% lidocaine with 2.5% epinephrine in adults is 7mg/kg. The patient weighs 110 lbs (or 50 kg) which allows her 350 mg or 35 ml (1% lidocaine has 10mg of lidocaine perml) of the anesthetic.

129
Q

129- Reticulate eythema is a side effect seen with which treatment?

A. Mesotherapy
B. Sclerotherapy
C. Diode laser
D. Cryotherapy
E. Pulsed dye laser

A

►C

Reticulate erythema is a recently-reported side effect of diode laser treatment. High energy fluences and a history of chilblains are believed to be predisposing risk factors. Laser treatment should be discontinued at the first sign of this complication.

130
Q

130 -The nasolabial two stage flap uses depends on which artery for flap survival:

A. Supratrochlear artery
B. Superior labial artery
C. Angular artery
D. Lateral Nasal artery
E. Inferior labial artery

A

►C

Angular artery provides blood supply for the nasolabial transposition flap.

131
Q

131- In a patient who is allergic to paraphenylinediamine, which of the following anesthetics should be avoided?

A. Bupivicaine
B. Etidocaine
C. Mepivicaine
D. Prilocaine
E. Benzocaine

A

►E

Paraphenyelenediamine is a common allergen found in permanent hair dyes. In individuals who are allergic to paraphenylendiamine, may also have allergic reactions to ester anesthetics, like benzocaine. The other answer choices are amide anesthetics. Other crossreactants to paraphenyelediamine include the preservative parabens, sulfonamides, and thiazide diuretics.

132
Q

132- Aging skin demonstrates all of the following characteristic changes except:

A. Loss of elasticity
B. Hypertrophy of sebaceous glands
C. Decreased number of Langerhans cells
D. Increase in subcutaneous fat
E. Decreased dermal collagen

A

►D

The loss and redistribution of subcutaneous fat is a characteristic finding of the aging face. The forehead, temporal fossae, malar cheeks and perioral region are the most commonly affected areas. Knowledge of senescent changes in fat distribution has altered the cosmetic surgeon‟s approach of

the aging face to one which includes augmentation rather than one of simply lifting and tightening the skin.

133
Q

133 -Which cosmetic injectable provides dermal augmentation through the harvesting of a patient”s fibroblasts?

A. Restylane
B. Isolagen
C. Hylaform
D. Juvaderm
E. Dermalogen

A

►B

Isolagen is an emerging technology whereby a patient”s own fibroblasts are isolated from a skin biopsy, reproduced and then re-introduced into the patient”s treatment site.

134
Q

134- Which of the following sterilization methods has the negative side effect of dulling instruments?

A. chemical autoclave
B. steam autoclave
C. dry heat
D. cold sterilization
E. gas sterilization

A

►B

Steam autoclave is the most common type of sterilization used in the office setting. It is not to be used with heat-sensitive plastics. A negative side effect of using steam autoclave is that it can dull sharp instruments.

135
Q

135 -The classic purpose for serrated scissors is:

A. For work on thicker tissues (back)
B. For work on delicate areas around the eyes and the ears
C. For gripping tissue while cutting
D. For tissue undermining
E. For sharp dissections of tissue

A

►C

Serrated scissors are used primarily for gripping tissue to prevent sliding while cutting.

136
Q

136- Which of the following is NOT a form of electrosurgery?

A. electrocautery
B. electrodesiccation
C. electrofulguration
D. electrocoagulation
E. electrosection

A

►A

Electrosurgery is a broad term to describe the use of electricity to produce thermal tissue damage. Electrocautery is a method of direct heat transfer to tissue by a heated element. No electric current is transferred.

137
Q

137- The most important function of a postoperative wound dressing is:

A. Provide a barrier to infection
B. Decrease tension on the wound
C. Create a moist environment
D. Prevent suture dehiscence
E. Immobilize the wound

A

►E

Providing hemostasis to a wound is the most important function of a wound dressing. This is particularly critical in the first few hours postoperatively when the vasoconstrictive effects of epinephrine are wearing off and the wound is at risk of bleeding or hematoma formation. The provision of a barrier to infection and creation of a moist, occlusive environment are also necessary functions of a dressing but are not as critical as immobilizing the wound and ensuring hemostasis.

138
Q

138 -What surgical dressing would you use if you wished to debride a wound, reduce pain, and provide a cooling effect?

A. Alginates
B. Foams
C. Hydrocolloid
D. Hydrogel
E. Films

A

►D

Hydrogels are 70-90% water, decrease pain, provide a cooling effect, and are useful for dry and necrotic wounds. Foams are suitable for use on light-to-medium exuding wounds. Hydrocolloids ensure the moist wound environment, promote the formation of granulation tissue, and provide pain relief by covering nerve endings with both gel and exudate. Large quantities of alginates are used each year to treat exudating wounds, such as leg ulcers, pressure sores, and infected surgical wounds.

139
Q

139- Which laser would be the best choice for the treatment of a red cosmetic tattoo around the lips?

A. Carbon dioxide
B. Pulsed dye
C. Q-switched ruby
D. Alexandrite
E. Nd:YAG

A

►C

Q-switched lasers can be effectively used to remove tattoos. However, immediate darkening of white, flesh colored, and pink/red tattoos has been reported after treatment with Q-switched lasers. Both ferric oxide and titanium dioxide (found in tattoo pigments) may be both be reduced after heating by the laser and produce dramatic darkening.

140
Q

140- Blue nodules in the skin are a potential complication of which cosmetic injectable?

A. Radiesse (hydroxyapatite)
B. Silicone
C. Restylane (hyaluronic acid)
D. Cosmoderm (collagen)
E. Sculptra (poly L-lactic acid)

A

►C

Although hyaluronic acid is believed to be inert in humans, rare reports exist of granulomatous foreign body reactions to this material. Skin biopsy of a Restylane nodule demonstrates multinucleated giant cells surrounding a blue amorphous material.

141
Q

141 -A patient has a 2.0 cm surgical defect on the left lower eyelid after having Mohs surgery. Which management option would be the least appropriate?

A. Primary closure
B. Secondary intention
C. Full-thickness skin graft
D. Tripier flap
E. Rhombic flap

A

►B

In lower eyelid reconstruction, tension should be oriented parallel to the lower eyelid to prevent the development of ectropion. Secondary intention would be the least appropriate management option in this location because normal wound contraction would increase the risk of ectropion.

142
Q

142- The use of dermabrasion to improve the cosmesis of a scar is best performed how long after the initial surgery?

A. 1 week
B. 3 weeks
C. 6 weeks
D. 6 months
E. 1 year

A

►C

Dermabrasion is the process of surgically planing or abrading the epidermis and dermis and is usually carried out with a rapidly rotating wire brush or diamond fraise. Following dermabrasion, reepithelialization from adnexal structures occurs with remodeling and replacement of collagen bundles in the papillary and reticular dermis. This can result in improvement of surgical or acne scars. When used in the post-operative period for surgical scars, dermabrasion is usually carried out 6 to 8 weeks following the procedure. Regional dermabrasions are routinel y carried out under local anesthesia. Post-procedure complications include scarring, pigmentary alterations, persistent erythema, and infection. Contraindications for dermabrasion include recent use of isotretinoin and recent facelift, browlift or other procedure involving extensive undermining. Reactivation of herpes labialis can be prevented in the lip area by administering antiviral prophylaxis.

143
Q

143 -A patient with a deep vertical frown line in the glabellar region requests Botox. Treatment will mostly target which one of the following muscles?

A. Oculi
B. Orbicularis oculi
C. Depressor superciliaris
D. Levator palpebrae superioris
E. Corrugator superciliaris

A

►E

Although the procerus and orbicularis oculi muscles are contributory, the development of vertical glabellar frown lines is mostly due to the overactivity of the corrugator supercilaris. These muscles function to pull the eyebrow inferomedially. procerus muscle contraction contributes largeley to the transverse rhytides along the nasal root. Orbicularis oculi contraction contributes to crow’s feet rhytides radiating from lateral canthal region. Depressor superciliaris muscle pulls the eyebrow inferomedially and partially contributes to vertical glabellar frown lines but not as much as the corrugators.

144
Q
  • All of the following are alpha-hydroxy acids except:

A. Lactic acid
B. Citric acid
C. Glycolic acid
D. Tartaric acid
E. Salicylic acid

A

►E

Alpha-hydroxy acids are naturally occurring carboxylic acids found in many foods. The alpha hydroxy acids include glycolic, lactic, malic, citric, and tartaric acids. Factors that determine the intensity of the peel include the concentration of the acid, pH, degree of buffering, vehicle formulation, frequency of application, conditions of delivery, and the length of time the acid is placed on the skin. Salicylic acid is a type of beta-hydroxy acid.

145
Q

145 -Which post-operative complication is shown in the photograph?

A. Trap door deformity
B. Hypertrophic scar
C. Ectropion
D. Temporal nerve paralysis
E. Eclabium

A

►C

Ectropion occurs when the eyelid margin turns outward away from the eye. In lower eyelid reconstruction, tension should be oriented parallel to the lower eyelid to prevent the development of ectropion.

146
Q

146 -Treatment of acne scarring would be most effective with which modality?

A. Pulsed dye laser
B. Intense pulsed light
C. Photodynamic therapy
D. Radiofrequency
E. Nd:YAG laser

A

►E

Treatment with the nonablative 1064-nm Q-switched Nd:YAG provides significant improvement in skin topography in patients with atrophic acne scars. The further improvements that are seen at 6- month follow-up suggest that dermal remodeling is a process that continues long after treatment.

Reference: Maluki AH, Mohammad FH. Treatment of atrophic facial scars of acne vulgaris by Q- Switched Nd:YAG (Neodymium: Yttrium-Aluminum-Garnet) laser 1064 nm wavelength. J Cosmet Laser Ther. 2012 Oct;14(5):224-33. PMID: 23016531

147
Q

147- The anatomic structure identified in the photograph is called the:

A. Nasal ala
B. Soft triangle
C. Columella
D. Philtral ridge
E. Nasal groove

A

►C

The columella is one of the cosmetic subunits of the nose. It is formed by the medial crura of the alar cartilage. The nasal ala forms the free margin on the lateral inferior portion of the nose and the superior aspect of the nostril. The soft triangle is located on the inferior nose and is bordered by the ala laterally, the nasal tip superomedially, and the columella inferomedially. The philtral ridge is a paired structure located on the upper cutaneous lip. The nasofacial sulcus separates the nasal sidewall from the medial cheek bilaterally.

148
Q

148- A surgeon plans to repair a defect on the lower lip with a wedge resection. All of the following statements are true regarding wedge resection of the lower lip except?

A. a V-shaped excision is often used
B. the vermillion border is marked prior to excision
C. the repair involves a two layered closure of skin and mucosa
D. up to 1/3 of the lip can be excised and closed primarily
E. the mental crease should not be crossed

A

►C

The lower lip has such great elasticity that full-thickness lesions that involve up to 1/3 of the lip can be excised and repaired primarily with excellent cosmetic and functional results. This type of repair

requires a three layered closure of mucosa, skin and muscle. Larrabee W. In: Principles of Facial Reconstruction. Lippincott-Raven: Philadelphia, 1995; 182-183.

149
Q

149 -This is used to describe a the setting for electrosurgery that is AC, bipolar, low voltage, and high amp:

A. Electrocaogulation
B. Hyfrecation
C. Electrosection
D. Electrolysis
E. Electrocautery

A

►A
Electrocaogulation is set on AC, bipolar, low voltage, and high amp. All the other ones do not fit the description.

150
Q

150 -When can a patient who undergoes follicular-unit hair transplantation expect hair growth of the grafted follicles to begin?

A. 7 to 10 days
B. 1 month
C. 2 to 3 months
D. 6 months
E. 9 months

A

►C

The use of follicular-unit grafts, which contain one to four hair follicles, represents the advancement in both surgical technique and aesthetic outcome in the field of hair transplantation. The grafted hair follicles typically begin to grow within 8 to 10 weeks of implantation and are expected to survive for the individual”s lifetime.

151
Q

151- Signs and symptoms of lidocaine toxicity include all of the following except:

A. Circumoral numbness
B. Ototoxicity
C. Slurred speech
D. Nystagmus
E. Seizure

A

►B

The first signs of lidocaine toxicity are CNS symtpoms that resemble inebriation with alcohol. These symptmoms include stupor, dysarthria, circumoral numbenss and dizziness. Further increases in toxicity leads to nausea, metallic taste, twitching, and seizures. Ototoxicity is not one of the toxic manifestations seen with lidocaine toxicity. Without epinephrine the maximum dose of lidocaine 4mg/kg. For a 70 kg individual, this is 300 mg or 30 ml of a 1% lidocaine solution. For a preparation of lidocaine with epinephrine the maximum dose is 7mg/kg. For a 70 kg individual, this is 500 mg or 50 ml of a 1% solution.

152
Q

152- To avoid injury to the facial nerve, undermining in the region of the temple should be performed at which anatomic level?

A. Dermal-epidermal junction
B. Reticular dermis
C. Subcutaneous fat
D. Above superficial fascia (SMAS)
E. Above the frontalis muscle

A

►C

In the region of the temple, lateral to the eyebrow, the temporal branch of the facial nerve lies in the superficial fascia. Once above the eyebrow, the nerve is located deep to t he frontalis muscle. In the region of the temple, it is therefore prudent to undermine in the superficial subcutaneous fat.

153
Q

153 -Paradoxical darkening has been associated with Q-switched ruby, Q-switched alexandrite, Q- switched Nd:Yag treatment of which tattoo color?

A. Green
B. Red
C. Purple
D. Black
E. Blue

A

►B

Paradoxic darkening of flesh-tone, red, and white tattoo inks with QSRL, Q-switched Nd:YAG, and Q-switched alexandrite lasers has been reported. The tattoo pigments most associated with this phenomenon are iron oxide and titanium dioxide. Ferric sulfide is converted to ferrous sulfate by the laser.

154
Q

154 -Which type of flap moves almost entirely in one direction in a linear fashion

A. Rhombic flap
B. Advancement flap
C. Rotation flap
D. Transposition flap
E. Bilobed flap

A

►B

An advancement flap is one that moves almost entirely in one direction or in a linear fashion. Rotation flap alters the direction of major closure tension forces. Transposition flap redirects and redistributes tension. Rhombic and bilobed flaps are both examples of transposition flaps.

155
Q

155- Jessner’s solution includes:

A. Lactic acid, alcohol, glycolic acid, resorcinol

B. Lactic acid, alcohol, glycolic acid, TCA
C. Lactic acid, alcohol, salicylic acid, glycolic acid
D. Lactic acid, resorcinol, salicylic acid, glycolic acid
E. Lactic acid, alcohol, salicylic acid, resorcinol

A

►E

Jessner’s is a superficial peeling agent, which may be used to treat dyschromia and texture abnormalities. TCA and glycolic acid are not ingredients in this formulation.

156
Q

156 -Which potential complications is seen exclusively with phenol peels?

A. Milia
B. Hypopigmentation
C. Contact dermatitis
D. Prolonged erythema
E. Laryngeal edema

A

►E

The principle concept of chemical peeling is to utilize a chemical agent to produce a controlled wound in the skin. Many different types of side effects can occur with chemical resurfacing and are typically related to the depth of the wound. Complications which are common to all peeling agents include: milia; acne; pigmentary changes; contact dermatitis; scarring; infection; prolonged erythema; textural changes and cold sensitivity. Unlike other peeling agents, phenol has the potential to cause severe adverse reactions which are exclusive to its use only. They include atrophy, cardiac arrhythmias and laryngeal edema.

157
Q

157 -The antiptosis subdermal suspension threads used in facial rejuvenation are composed of:

A. Poly-L-lactic acid
B. Polyglactin
C. Polypropylene
D. Polydioxanone

E. Polyglycolic acid

A

►C

The Aptos subdermal suspension thread technique is a minimally invasive procedure which targets the ptotic changes seen with facial aging. 2-0 and 3-0 polypropylene threads are tunneled in the dermis to lift and suspend the skin and subcutaneous tissue.

158
Q

158 -Alopecia neoplastica is most commonly associated with which malignancy?

A. Lymphoma
B. Squamous cell carcinoma
C. Breast carcinoma
D. Colon carcinoma
E. Melanoma

A

►C

While various malignancies have a predilection for metastasis to the scalp where they can cause hair loss, alopecia neoplastica is most commonly associated with metastatic breast carcinoma.

159
Q

159- In organ transplant recipients, all of the following factors increase susceptibility to the development of skin cancer except:

A. Fair skin (Fitzpatrick types I-III)
B. History of chronic sun exposure
C. Duration of immunosuppression
D. History of HPV infection
E. CD8 lymphocytopenia

A

►E

All organ transplant recipients are at increased risk for the development of cutaneous malignancies. However, the above-mentioned factors, with the exception of choice E, place these individuals at further risk. CD4 lymphocytopenia, rather th an CD8, is another identified risk factor as is older age, history of actinic keratoses and history of skin cancer.

160
Q

160- A Z-plasty is performed to:

A. Decrease tension on a wound
B. Obtain wound eversion
C. Shorten the length of an excision
D. Correct dog-ears
E. Make use of excess tissue

A

►A

A Z-plasty is a transposition flap used most commonly in the treatment of contractures and scars. The main advantages to performing a Z-plasty are to decrease the tension on a wound, change the orientation of a scar, and camouflage a scar by breaking it up into smaller components.

161
Q

161- Which of the following sutures is best used for mucous membranes?

A. Catgut
B. Silk
C. Polyglactin 9, 10
D. Nylon
E. Polydioxanone
c

A

►B

Silk is a natural nonabsorbable suture used mainly for eyelid, lip, or intraoral surgeries as they are more comfortable to the patient on these surfaces. This suture has excellent handling as it is less stiff and flows easily through soft tissue. However, it has high tissue reactivity, low tensile strength, and is associated with a higher risk of infection.

162
Q

162- On average, how much does a full-thickness graft contract when removed from a donor site?

A. 15%
B. 30%
C. 50%
D. 60%
E. 80%

A

►A

On the average, a full-thickness graft may contract by about 15% when removed from a donor site.

163
Q

163 -Pre-testing for allergy to collagen is required for which cosmetic filler?

A. Radiance
B. Artecoll
C. Perlane
D. Poly L lactic acid
E. Silicone

A

►B

Artecoll is a non biodegradable injectable filler composed of microspheres of polymethylmethacrylate which are suspended in bovine collagen. As with other types of injectable bovine collagen, patients must be tested for allergy to bovine collagen prior to treatment.

164
Q

164 -Which tattoo pigment has most commonly been associated with phototoxic reactions?

A. Titanium dioxide
B. Mercuric sulfide
C. Carbon
D. Iron oxide
E. Cadmium sulfide

A

►E

Cadmium sulfide produces a yellow tattoo. Phototoxic reactions are most commonly caused by cadmium sulfide tattoo pigment. Red tattoos have also been associated with phototoxic tattoo reactions. However, this is thought thought to be due to the addition of cadmium sulfide which enhances and brightens the red color.

165
Q

165 -Which of the following is true regarding a Baker-Gordon peel?

A. 88% phenol is the most important component for efficacy
B. It contains salicytic acid
C. It contains resorcinol
D. It is considered a medium depth peel
E. Croton oil is considered the most important component for efficacy

A

►E
Croton oil is considered the most important component for efficacy. It is a epidermolytic that enhances penetration. A Baker Gordon peel is a deep peel that contains 88% phenol, tap water, croton oil, and septisol.

166
Q

166 -All of the following statements are true regarding nerves blocks on the face except:

A. Anesthesia is placed around a nerve trunk
B. Lidocaine with epinephrine may be used
C. Numbness occurs in areas other than the operative site
D. Tissue distortion is minimized
E. Hemostasis is achieved using epinephrine

A

►E

While regional nerve blocks offer many advantages in cutaneous surgery, hemostasis is not one of them. Since the anesthetic is injected distant to the operative site, the vasoconstrictive effects of the epinephrine, and hence hemostasis, are not provided at the surgical site.

167
Q

167 -Which statement is true regarding relaxed skin tension lines (RSTLs)?

A. they occur as a result of increased elastic tone
B. they lie parallel to the underlying muscles
C. the long axis of a wound often lies in the direction of the RSTL
D. incisional scars should be placed perpendicular to RSTLs
E. smiling minimizes the appearance of RSTLs

A

►C

Relaxed skin tension lines are creases in the skin that are present at rest and develop as a consequence of the decreased elasticity that occurs with aging and solar damage. These lines lie perpendicular to the underlying musculature and can be accentuated by smiling or frowning. An undermined circular wound will often form an oval shape due to muscular tension, and will have its long axis oriented within these relaxed tension lines. Knowledge of RSTLs is important in cutaneous surgery because placing incisions within these lines will ensure the most favorable cosmetic result.

168
Q

168 -Which needle shape is most commonly used in cutaneous surgery?

A. 1/4 circle
B. 3/8 circle
C. 1/2 circle
D. 5/8 circle
E. compound curve

A

►B

Needles are either straight or curved. Curved needles have their curvature described either as a fraction of a circle or a compound curve. The greater the fraction of a circle, the more pronation and supination of the wrist required by the surgeon to place the needle. The 3/8 circle needle is easy to use in large, superficial areas and is the most commonly used needle for cutaneous surgery.

169
Q

169- Sensory innervation of the glabella and mid-forehead is provided by which nerve?

A. infratrochlear nerve
B. supraorbital nerve
C. supratrochlear nerve
D. temporal branch of the facial nerve
E. zygomatic branch of the facial nerve

A

►C

The supratrochlear, supraorbital and infratrochlear nerves are branches of the ophthalmic division of the trigeminal nerve, supplying sensory innervation to the face. The supratrochlear nerve exits the orbit approximately one centimete lateral to the midline and traverses upward along the medial supraorbital rim along the supratrochlear notch. It provides sensory innervation to the medial upper eyelid and glabella, the medial anterior forehead, and the scalp. The supraorbital nerve supplies the lateral forehead and scalp as well as the upper eyelid. The infratrochlear nerve provides sensory innervation to the root of the nose and to portions of the medial canthus. The facial nerve provides motor innervation to muscles of the face. The temporal branch of the facial nerve supplies motor innervation to the upper orbicularis oculi and the frontalis muscle, as well as the anterior and superior auricular muscles. The zygomatic branch of the facial nerve provides innervation to the inferior portion of the orbicularis oculi, zygomaticus minor and zygomaticus major muscles, as well as levator labii superioris.

170
Q

170- True statements regarding skin cancer in organ transplant recipients include all of the following except:

A. 65 fold increase in development of SCC compared with general population
B. Mohs micrographic surgery indicated for in-transit metastases
C. Cutaneous malignancies develop 3-5 years after organ transplantation
D. Extent of tumor development related to degree of immunosuppression
E. Skin cancer is the most common cancer in transplant patients

A

►B

In-transit metastases is a common manifestation of metastatic disease.

171
Q

171 -Which type of collagen is the first to be deposited in a healing wound?

A. Type I collagen
B. Type II collagen
C. Type III collagen
D. Type IV collagen
E. Type VII collagen

A

►A

Coagulation/Inflammation, tissue formation, and scar/remodeling are the three phases important in wound healing. Collagen deposition occurs during the second phase. Fibroblasts migrate into the wound along the fibrin-fibronectin matrix deposited in the initial clot. The fibroblasts subsequently produce type I and III collagen, elastin, and proteoglycans. Collagen type III is the predominant collagen in early wound healing.

172
Q

172 -The cosmetic subunits are designated based upon their similarity in all of the following except:

A. Texture
B. Color
C. Sebaceous features

D. Hair density
E. Eccrine features

A

►E

The cosmetic unit are based on their similarity in texture, color, solar exposure, hair density, and sebaceous features. They are not from eccrine features.

173
Q

173- Which of the following cosmetic injectables does not illicit an inflammatory response?

A. Radiesse (hydroxyapatite)
B. Artecoll (polymethylmethacrylate)
C. Silicone
D. Zyplast (collagen)
E. Sculptra (poly L-lactic acid)

A

►A

Radiesse is an injectable, biodegradable filler composed of calcium hydroxyapatite microspheres. It is a normal constituent of bone and is therefore biocompatible. Current studies are examining the role of hydroxyapatite in augmenting the craniofacial skeleton.

174
Q

174- The „snap test‟ is performed prior to surgery at which anatomic location?

A. Hand
B. Ear
C. Mouth
D. Eyelid
E. Neck

A

►D

The snap test determines the laxity of the lower eyelid. It should be performed prior to surgery, laser resurfacing or Botox injections of the lower eyelid to assess the potential for ectropion formation. Ratner D et al. Cutaneous laser resurfacing. J Am Acad Dermatol 1999; 41(3):365389.

175
Q

175 -Which of the following local anesthetics has the longest duration of action?

A. Lidocaine
B. Bupivacaine
C. Procaine
D. Benzocaine
E. Mepivacaine

A

►B

Bupivacaine is an amide type of local anesthetic. Its duration of action is approximately 3-5 hours and is the longest one listed. Following bupivacaine is mepivacaine, lidocaine, and procaine. The duration of action is based on the amine portion of the molecule.

176
Q

176- Chlorohexidine should not be used around the following area:

A. Eyes
B. Nose
C. Forehead
D. Arms
E. Feet

A

►A

Chlorhexidine or hibiclens can cause keratitis and allergic reaction if used directly in contact with the eye. It can cause ocular keratitis. It can also cause tympanic membrane damage.

177
Q

177 -Healing by secondary intention will offer the most favorable cosmetic result at which location?

A. Nasal tip
B. Malar cheek
C. Chin
D. Forehead
E. Medial canthus

A

►E

Indications for healing of full-thickness skin wounds by secondary intention include infected wounds, patients who refuse reconstructive surgery or who are poor surgical risks for reconstructive surgery, and resection of tumors with high rates of recurrence. In addition, second intention, or granulation, can be utilized when the cosmetic result is anticipated to be superior or equal to reconstruction. Disadvantages of healing by granulation include prolonged wound healing time and somewhat unpredictable cosmetic results. Anatomic sites lending themselves best to second intention healing, with excellent results, are the concave areas on the face. These include the medial canthus, the nose-cheek junction, the auricular concha, the preauricular cheek, and the retroauricular scalp. The nasal tip, malar cheek, chin and forehead are all convex surfaces on the face.

178
Q

178- Which porphyrin is targeted in aminolevulinic acid-photodynamic therapy?

A. Protoporphyrin IX
B. Uroporphyrinogen
C. Coproporphyrin III
D. Coproporphyrinogen
E. Porphobilinogen

A

►A

Protoporphyrin IX is the substrate for the final rate-limiting step of heme synthesis. This enzymatic reaction is catalyzed by ferrochelatase. The preferential uptake of aminolevulinic acid by cancerous and pre-cancerous cells results in higher concentrations of protoporphyrin IX. This can then be selectively targeted by photodynamic therapy.

179
Q

179- Which tattoo pigment has been most commonly associated with pseudolymphomatous reactions?

A. Mercuric sulfide
B. Cadmium sulfide
C. Phthalocyanine dyes
D. Malachite
E. Cobalt

A

►A

Some delayed type hypersensitivity reactions may lead to pseudolymphoma, most commonly caused by red tattoo pigment (mercuric sulfide).

180
Q

180 -A patient undergoing Mohs surgery undergoes a rotation flap from the lateral canthus/cheek to close an infraorbital defect. What is the name of this flap?

A. Rieger flap
B. Mustarde flap
C. O to Z
D. Bilobed
E. Abbe flap

A

►B

This patient had a Mustarde flap, a rotation flap from the lateral canthus or cheek that close the lower eyelid or infraorbital defect. A Rieger flap is a dorsal nasal rotation flap with backcut at the glabella; it is also knows as a hatchet flap or glabellar turn-down flap. O to Z is a double rotation flap. A bilobed is a transposition flap most often used on the nose. Finally, and Abbe flap, or a lip- switch flap, is a lower lip flat of mucosa, muscle, skin, and subucutis that is transposed superiorly to the repair an upper lip defect based on inferior labial artery blood supply.

181
Q

181 -Which of the following immunosuppressive agents may exert a protective effect against the development of skin cancers in organ transplant patients?

A. Cyclosporine
B. Rapamycin
C. Tacrolimus
D. Corticosteroids
E. Azathioprine

A

►B

Rapamycin (also called sirolimus) is a macrolide antibiotic and a structural analog of FK 506. It is a potent immunosuppressive agent which inhibits mTOR (a member of P13K family kinases). Despite its immunosuppressive effects, preliminary data show a decreased incidence of skin cancer in organ transplant patients treated with rapamycin and postulate that it may exert a protective effect against cutaneous malignancies.

182
Q

182 -The relaxed skin tension lines overlying the scapula are:

A. Do not exist over the scapula
B. Oriented vertically
C. Oriented circumferentially
D. Oriented horizontally
E. Perpendicular to the underlying muscle groups

A

Relaxed skin tension lines (RSTLs) are creases in the skin that are present at rest and develop as a consequence of the decreased elasticity that occurs with aging and solar damage. These lines lie perpendicular to the underlying musculature. They tend to run diagonally on the upper back whereas on the central trunk, they are typically circumferential. Knowledge of RSTLs is important in cutaneous surgery because placing incisions within these lines will ensure the most favorable cosmetic result.

183
Q

183- While removing sutures from an excision performed on the lateral neck, your patient reports decreased sensation of the helix and lobule of the ipsilateral ear. Which of the following nerves was likely injured during surgery?

A. Auriculotemporal nerve
B. Lesser occipital nerve
C. Auricular branch of vagus nerve
D. Greater auricular nerve
E. Spinal accessory nerve

A

►D

Sensory innervation of the ear is provided by the greater auricular nerve, the auriculotemopral nerve, the lesser occipital nerve and the auricular branch of the vagus nerve. The greater auricular nerve is composed of the C2 and C3 branches of the cervical plexus and lies in the posterior triangle of the neck. It provides sensation to the helix, antihelix, antitragus, posterior auricle and the lobule. The spinal accessory nerve is a motor nerve which innervates the trapezius muscle. This nerve is also subject to injury in the lateral neck but would cause motor deficits rather than sensory deficits.

184
Q

184 -The type of repair depicted in the photograph is a:

A. Rhombic flap
B. Nasolabial flap
C. Rotation flap
D. Full-thickness skin graft
E. Island pedicle flap

A

►E

The island pedicle flap is a type of advancement flap commonly used for medial ch eek defects. The closure can often be camouflaged in the melolabial fold.

185
Q

185 -Full thickness skin grafts are more likely than split thickness skin grafts to demonstrate:

A. Alopecia
B. Contraction
C. Necrosis
D. Hypohidrosis
E. Hypopigmentation

A

►C

Full thickness skin grafts have more adnexal structures, vessels, and structures to supply. Therefore, they are more prone to necrosis. However, they are less prone to contraction, alopecia, hypohidrosis, and hypopigmentation for similar structural reasons.

186
Q

186 -The use of topical vitamin K has been shown to:

A. Decrease the appearance of infraorbital pigmentation
B. Minimize the appearance of telangiectasia
C. Reduce the severity of laser-induced purpura
D. Increase collagen production
E. Induce keratinocyte differentiation

A

►C

Treatment of benign vascular lesions with the pulsed dye laser often produces significant postoperative purpura. Topical vitamin K has been shown to decrease the severity of laser induced purpura although its mechanism of action has yet to be determined. No other cosmetic effects of topical vitamin K have been proven to be statistically significant. Topical vitamin K has not been show to reduce the apperance of pigmentation or telangiecatasia in any meaningful way. Additionally, it has no demonstrable effect on collagen synthesis or kerainocyte differentiation.

187
Q

187 -What part of the eye may be damaged by exposure to irradiation from the carbon dioxide laser?
A. Retina
B. Lens
C. Sclera
D. Cornea
E. Iris

A

►D

The carbon dioxide laser operates at a wavelength of 10,600 nm and targets water as a chromophore. Because of the high water content of the cornea, it may be damaged by exposure to irradiation from the carbon dioxide laser. Exposure to the erbium may cause corneal damage as well.

188
Q

188- The rhytids identified in the photograph are referred to as:

A. Marionette lines

B. Relaxed skin tension lines
C. Crow‟s feet
D. Nasolabial folds
E. Bunny lines

A

►E

Bunny lines are located on the dorsum of the nose and are formed by the contraction of the nasalis muscle. Marionette lines form at the lateral oral commissures and run inferolaterally toward the jawline just over and lateral the region of the depressor anguli oris muscle. Crows feet lines radiate from the lateral canthal region and are caused by the orbicular oculi contraction. Nasolaial foles run from the apical triangle of the upper cutaneous lip down lateral to the oral commissures and are caused by dynamic facial muscle movements and fat pad atrophy over time. Relaxed skin tension lines run perpendicule to underlying muscle orientation.

189
Q

189- The Shaw scalpel is used:

A. As a form of electrocautery
B. To enhance cyst wall removal
C. For incision and drainage
D. For thicker tissue (back)
E. For delicate work around the eyes and ears

A

►A

The Shaw scalpel provides the ability to cut through tissue with minimal blood loss. This instrument employs an elctronically heated metal cutting balde that heats up while it cuts through tissue, and coagulates fine vessels when they are divided. This instrument has been used frequently for treatment of rhinophyma and in parotid gland surgeries.

190
Q

190 -Cosmetic units are defined by each of the following anatomic boundaries except:

A. Cheek

B. Glabellar region
C. Nasofacial sulcus
D. Philtral ridge
E. Nasolabial folds

A

►A

The term cosmetic unit is used to define regions of the face by specific characteristics such as color, texture, presence of hair, and prominence of sebaceous glands. In dermatologic surgery, it is preferable to conceal incisions within the boundary lines of cosmetic units. In addition, moving skin from one cosmetic unit to another should be avoided when possible to minimize the appearance of apposed skin of dissimilar quality.

191
Q

191 -The “trapdoor effect” that can occur with a flap can be avoided by which of the following techniques:

A. Wide undermining
B. Using additional sutures
C. Performing the flap in a two staged procedure
D. Using a flap with length to width ratio of less than 3:1
E. Avoid thinning the flap

A

►A

Wide undermining of the primary defect can release tension and prevent tenting or the trap door deformity of the flap.

192
Q

192- A patient has a 1.5cm surgical defect on the right nasal sidewall that reveals the lateral cartilage. Which management option would be the least appropriate?

A. Primary closure
B. Secondary intention
C. Full-thickness skin graft

D. Cheek transposition flap
E. Forehead flap

A

►C

Full-thickness skin grafts (FTSGs) depend upon a viable, vascular bed at the recipient site in order to survive. Exposed cartilage is an avascular tissue and therefore would not be able to support the metabolic requirements of a FTSG.

193
Q

193 -Which of the following treatment modalities for basal cell carcinoma has the highest long term cure rate?

A. MOHS surgery
B. Radiation
C. Cryotherapy
D. Electrodessication and curettage
E. Surgical excision

A

►A

Five year cure rates of basal cell carcinomas (BCC) treated with MOHS is about 99%. The other options listed range from 90-92% five year cure rates.

194
Q

194 -Each of the following are branches of the internal carotid artery except:

A. Ophthalmic artery
B. Supratrochlear artery
C. Dorsal nasal artery
D. Angular artery
E. Supraorbital artery

A

►D

The angular artery is a branch of the external carotid artery. It arises from the facial artery and courses superiorly along the nasofacial angle until it reaches the area of the medial canthal tendon. At this location, the angular artery anastamoses with the dorsal nasal branch of the ophthalmic artery establishing a communication between the internal and external carotid arterial systems.
In regards to dermatologic surgery, it is important to be aware that the internal carotid artery branches into the supraorbital, supratrochlear, infratrochlear, and dorsal nasal arteries to provide blood to the central face.

195
Q

195 -The following flaps are types of transposition flaps except:

A. Rhombic flap
B. Nasolabial
C. Bilobed
D. A to T
E. Z plasty

A

►D

The Z plasty, Bilobed, Rhombic, nasolabial flaps are all examples of transposition flaps. In each of these, the flap is transposed over a normal piece of skin to be placed in the recipient site. Whereas, the A to T flap, is an advancement flap, in which tissue is moved in a linear direction to cover the defect.

196
Q

196- Compared to intact skin, what is the tensile strength of a wound 6 months after surgery?

A. 30%
B. 50%
C. 70%
D. 90%
E. 100%

A

►C

Tissue remodeling allows the host to develop a scar that is approximately 70% of the original strength of the skin.

197
Q

197 -A patient with end stage liver disease comes to your surgical practice. Which of the following anesthetics should be used with this patient?

A. Benzocaine
B. Etidocaine
C. Lidocaine
D. Mepivicaine
E. Prilocaine

A

►A

Benzocaine, an ester, should be used as it is the only anesthetic mentioned above that is metabolized by plasma cholineseterase and then is excreted by the kidney and not the liver.

198
Q

198- Preliminary studies with deoxycholic acid demonstrate efficacy in the treatment of:

A. Superficial rhytids
B. Submental fat
C. Soft tissue augmentation
D. Postinflammatory hyperpigmentation

E. Acne scarring

A

►B

Deoxycholic acid is a detergent that dissolves fat. It is ATX-101.

199
Q

199- Which of the following suture materials is most likely to cause infection?

A. Silk
B. Vicryl (polyglactin)
C. Prolene (polypropylene)
D. Nylon
E. PDS (polydioxanone)

A

►A

Silk is a nonabsorbable braided suture which has been shown to aid in the production of infection. It is believed that the interstices of braided or twisted suture materials enhance the potential for developing infection by providing privileged sites which harbor bacteria.

200
Q

200- Formation of granulomas is a potential complication of treatment with:

A. Isotretinoin
B. Autologous fat
C. CO2 resurfacing
D. Silicone
E. Hydroxychloroquine

A

►D

Silicone is a synthetic, viscous compound which is composed of long polymers of dimethylsiloxanes. Silikon-1000 is one silicone product which is available in the United States and

is FDA-approved for the ophthalmic treatment of complicated retinal detachment. The number 1000 refers to the product”s viscosity which is measured in centistokes. As a reference, water has a viscosity of 100 centistokes and mineral oil has a viscosity of about 350 centistokes. With respect to cosmetic applications, silicone is not FDA-approved for soft tissue augmentation. In addition to the occurrence of hypersensitivity reactions and product migration, there have been many reports of granuloma formation after silicone injection, even many years post-treatment.

201
Q

201 -What nerve innervates the skin between the 1st and 2nd toes?

A. saphenous nerve
B. posterior tibial nerve
C. sural nerve
D. superficial peroneal nerve
E. deep peroneal nerve

A

►E
deep peroneal nerve innervates the skin between the 1st and 2nd toes. The saphenous innervates the instep and medial ankle. The posterior tibial innervates the heel and middle sole of the foot. The sural nerve innervates the 5th toe and lateral side of the sole. The superficial peroneal nerve innervates the skin of the toes other than the outside of the 5th toe and in between the 1st and 2nd toes.

202
Q

202 -Injection of Botox at the location identified in the photograph would denervate which muscle?

A. Orbicularis oculi
B. Frontalis
C. Corrugator superciliaris
D. Procerus
E. Levator palpebrae superioris

A

►D

The development of vertical glabellar frown lines is due to the overactivity of the orbicularis oculi, procerus and corrugator supercilaris muscles. These muscles function to pull the eyebrow inferomedially.

203
Q

203 -Which of the following complications has been reported with infraorbital injections of Botox?

A. Nystagmus
B. Blindness
C. Festooning
D. Astigmatism
E. Photophobia

A

►C

Festooning of the lower eyelid is a recently reported complication of infraorbital Botox injections. The affected patient had a prior lower lid blepharoplasty which likely weakened the orbicularis oculi muscle. The author postulates that festooning resulted from further muscle weakness due to Botox treatment.

204
Q

204-Which structual component of local anesthetics (e.g. lidocaine) is responsible for the onset of activity?

A. Aromatic ring
B. Intermediate chain
C. Amine end
D. Length of the carbon chain
E. None of the above

A

►A

Local anesthetics like lidocaine contain 3 principle structural elements. The aromatic ring determines the onset of activity, the intermediate chain defines the class (amide vs. ester), and the amine end is responsible for the duration of action.

205
Q

205 -Botulinum toxin type A cleaves which protein in the presynaptic neuron?

A. Acetylcholine
B. Snap-25
C. Serotonin
D. Synaptobrevin (VAMP)
E. Syntaxin

A

►B

Botulinum toxin type A is FDA approved for the treatment of glabellar wrinkles. Botulinum toxins act by a three step process of binding, internalization by receptor medicated endocytosis and enzymatic activation. It has specific light chain intracellular binding sites and different sites of action on different SNARE (synaptosomal associated protein receptor [SNAP]) proteins. The SNARE proteins are intimately involved in releasing acetylcholine at presynaptic terminals. Botulinum toxin A cleaves SNAP-25 whereas B cleaves synaptobrevin.

206
Q

206 -Which of the following criteria carries the worst prognosis for a patient with a squamous cell

carcinoma?

A. Size of tumor > 1 cm
B. Depth of invasion > 4 mm
C. Perineural invasion
D. Anatomic location
E. Immunosuppression

A

►B

Many factors contribute to the aggressive behavior of cutaneous squamous cell carcinoma. For example, tumors in immunosuppressed patients and tumors with the histologic subtypes desmoplastic and acantholytic are often biologically aggressive squamous cell carcinomas. Compared with other risk factors, depth of invasion >4 mm and size > 2cm demonstrate the greatest risk for metastasis.

207
Q

207 -The cutaneous lip and chin are divided into cosmetic units by which anatomic boundary?

A. Vermillion border
B. Nasolabial fold
C. Marionette lines
D. Mental crease
E. Philtral ridge

A

►D

The term cosmetic unit is used to define regions of the face by specific characteristics such as color, texture, presence of hair, and prominence of sebaceous glands. In dermatologic surgery, it is preferable to conceal incisions within the boundary lines of cosmetic units. The mental crease, as demonstrated in the photograph, divides the cutaneous lip and chin.

208
Q

208- A 78 year old man with an infiltrative basal cell carcinoma of the left nasal ala. what is the best treatment option?

A. MOHS surgery
B. 5-fluorouracil cream
C. Electrodessication and curettage
D. Standard excision
E. Imiquimod

A

►A

Based on the histopathologic classification and location, this infiltrative basal cell carcinoma (BCC) is best treated with MOHS surgery to preserve normal tissue and structure. 5-fluorouracil cream is used to treat superficial BCC, squamous cell carcinoma in situ (SCCIS), and actinic keratoses. Electrodessication and curettage is used to treat SCCIS and superficial BCC. Standard excision would be possible if preservation of normal tissue and structure were not as crucial as in this area, the nasal ala. Imiquimod is used to treat actinic keratoses and superficial basal cell carcinoma.

209
Q

209- Which of the following facial rejuvenation techniques creates microthermal zones?

A. Radiofrequency
B. Ablative CO2 laser
C. Intense pulsed light
D. Dermabrasion
E. Fractional photothermolysis

A

►E

Fractional photothermolysis utilizes an infrared laser to create thousands of microthermal zones with normal skin remaining between these zones. The zones of normal skin support rapid reepithelialization and result in much faster healing times than with traditional resurfacing procedures.

210
Q

210- All of the following are considered the danger zones of the face except:

A. Frontal branch of the temporal artery at the temple

B. Facial artery as it crosses the mandibular rim
C. Angular artery as it courses near the nose
D. Posterior auricular artery runs near sulcus dividing mastoid from the posterior ear
E. Anterior aspect of the jaw where the angular artery crosses the jaw

A

►E

All of the following are considered danger zones of the face except for the anterior aspect of the jaw where the angular artery crosses the jaw. The angular artery is most dangerous when it courses near the nose.

211
Q

211 -Which ocular structure is at most risk of injury from an erbium: YAG laser?

A. Lens
B. Retina
C. Cornea
D. Vitreous humor
E. Iris

A

►C

Injury from lasers may occur via direct or indirect ocular exposure. Damage is generally wavelength specific. Laser that target hemoglobin or pigment may cause damage to the retinal pigment or vasculature while lasers that target water as a chromophore (carbon dioxide and erbium) can damage the cornea.

212
Q

212- A patient who had liposuction 5 weeks ago presents with multiple firm nodules at the cannula insertion sites. You suspect the diagnosis is:

A. Foreign body granuloma
B. Herpes simplex infection
C. Organized hematoma
D. Mycobacterial infection

E. Cold panniculitis

A

►D

Atypical mycobacterial infections are occurring with increasing frequency after cosmetic surgery procedures. These infections typically occur 4-14 weeks after a procedure as a late-occurring complication. Firm nodules at the treatment site or dehiscence of a previously healed wound may be presenting signs of atypical mycobacterial infection.

213
Q

213- The tissue temperature required for destruction of basal cell or squamous cell skin cancers with cryotherapy:

A. -40 to 50” C
B. 0 to -10 C
C. 10 to 20 C
D. -60 to- 70 C
E. -10 to -20 C

A

►A

-40 to -50 “C is the temperature required in the treated skin to effectively destroy basal cell and squamous cell skin cancers. To treat malignant skin lesions a thermocoupling device can be used to measure the exact temperature of the skin while freezing. After anesthetizing the skin, this device is inserted into the estimated depth of the skin lesion. Cryotherapy is then performed until the thermometer reaches a temperature of approximately -50 degrees Celsius. Keratinocytes need to be frozen to -50 degrees Celsius for effective destruction, while melanocytes are destroyed at 5 degrees Celsius. To treat cutaneous malignancies,-50 Degress Celsius is the effective temperature. For benign skin lesions such as verrucae, -20 degrees Celsius is effective.

214
Q

214- All of the following are branches of the internal carotid artery except:

A. Ophthalmic artery
B. Angular artery

C. Supraorbital artery
D. Dorsal nasal artery
E. Supratrochlear artery

A

►B

Branches of the internal carotid artery include the ophthalmic artery, supraorbital artery, dorsal nasal artery, supratrochlear artery, palpebral artery, and lacrimal artery. Branches of the external carotid artery include the superficial temporal artery, facial artery (inferior labial, superior labial, angular branches), maxillary artery, posterior auricular, and occipital artery. The external and internal carotid arteries anastamose in the periorbital region.

215
Q

215 -Which of the following is the major antioxidant in the human epidermis?

A. Superoxide dismutase
B. Coenzyme Q10
C. Ascorbic acid
D. Glutathione
E. Alpha-tocopherol

A

►E

Research into the preventive role of topical antioxidants in photoaging is based on the free radical theory of aging. All of the choices above are antioxidants which are naturally found in the skin, however, alpha-tocopherol is the major antioxidant in human epidermis. Epidermal depletion of this vitamin has been shown to be an early and sensitive marker of environmental oxidative damage.

216
Q

216- Which of the following histochemical stains can be used intraoperatively during Mohs micrographic surgery to confirm the diagnosis of extramammary Paget’s disease?

A. Neuron-specific enolase
B. Toluidine blue
C. S-100

D. Cytokeratin 7
E. Myeloperoxidase

A

►D

Cytokeratin 7 is a structural component of the cytoskeleton that stains poorly differentiated tumors of the epithelium. This low molecular weight cytokeratin positively stains Paget cells. Neuron- specific enolase stains merkel cell carcinoma, toluidine blue stains mast cells and myeloperoxidase stains cells with myeloid differentiation such as leukemia cells. S -100 protein is a non-specific stain that is commonly used as an adjunctive marker in the diagnosis of melanoma.

217
Q

217 -A 40 year-old woman from Southern California has wrinkles at rest on her forehead, scattered telangiectasia on her nose and a few seborrheic keratoses on her chest. What category in Glogau’s photoaging classification scale does this patient represent?

A. Type 1
B. Type 2
C. Type 3
D. Type 4
E. Type 5

A

►C

Glogau‟s photoaging scale is a classification system that employs clinical markers of photodamage to determine an individual‟s level of photoaging. The scale ranges from Type 1 through Type 4, with Type 4 demonstrating the most extensive photodamage. Type 3 on the Glogau scale is classified as advanced photoaging and describes a patient who is typically 50 years of age or older, has wrinkles at rest, telangiectasias, obvious dyschromias and visible keratoses.

218
Q

218 -What is the term used to describe the parallel nature of coherent light waves is?

A. monochromicity
B. coherence

C. collimation
D. energy
E. power

A

►C

Collimation is the parallel nature of coherent light waves. Monochromaticity is the emission of a single wavelength. Coherence is the term for light waves traveling in phase. Energy is the fundamental unit of work. Power is the rate of energy delivery measured in watts.
c

219
Q

219- Which of the following cosmetic injectables is FDA-approved for the treatment of lipoatrophy?

A. Radiance
B. Silicone
C. Botox
D. Sculptra
E. Isolagen

A

►D

Sculptra (called New-Fill outside of the US) is a biodegradable filler composed of poly-L-lactic acid, the same material used in absorbable sutures. It is biocompatible and nonallergenic and was approved by the FDA in August 2004 for the treatment of HIV-associated lipoatrophy.

220
Q

220 -Which of the following is true regarding lidocaine:

A. 1% lidocaine is equal to 1g/10ml
B. Duration with no epinephrine is 4-6 hours
C. Maximum dose with no epinephrine is 7mg/kg
D. Beta blockers increase lidocaine levels
E. Allergy most commonly occurs to propylene glycol preservatives

A

►D

Beta blockers increase lidocaine levels. The rest of the answers are false. 1% lidocaine is equal to 1g/100ml or 10mg/ml. Duruation of lidocaine with no epi is 30-60 minutes. Maximum dose of lidocaine with no epi is 4.5 mg/kg, with epi it is 7 mg/kg. Allergy to lidocaine is most commonly due to paraben preservatives.

221
Q

221 -Which can work in a wet field?

A. Electrocautery
B. Electrodessication
C. Electrofulguration
D. Electrosection
E. Electrocoagulation

A

►A

Electrocautery is the only one to work in a wet field as it has no current and works via heat.

222
Q

222- Which of the following is the earliest symptom of lidocaine toxicity:

A. Tachycardia
B. Perioral tingling
C. Nystagmus
D. Cyanosis
E. Seizure

A

►B

The maximum dosage of lidocaine is 4.5 mg/kg without epinephrine and 7.0 mg/kg with epinephrine. Signs of lidocaine toxicity start at 1-5 microgm/ml with increased anxiety, talkativeness, tinnitus, numbess/tingling around lips, metallic taste, double vision. Higher levels of toxicity may cause nystagmus, muscle twitching, tremor and finally seizures and respiratory arrest.

223
Q

223- Activation of the procerus muscle causes

A. Periocular wrinkles
B. Wrinkles at the nasal root
C. Wrinkles on the forehead
D. Perioral wrinkles
E. Accentuation of the melolabial folds

A

►B

The procerus muscle causes wrinkling at the nasal root and is often targeted with Botox therapy for improved cosmesis.

224
Q

224- Which nerve provides the sensory innervation to the tragus?

A. Auricular branch of vagus
B. Facial
C. Glossopharyngeal
D. Auriculotemporal
E. Greater auricula

A

►D

The auriculotemporal nerve which is a branch of the mandibular branch of the trigeminal nerve innervates the tragus. The sensory innervation to the auricle is provided by the greater auricular nerve, and to a lesser extent the lesser occipital nerve. The external auditory canal, concha, and posauricular sulcus supplied variably by 3 nerves - the auricular branch of the vagus (CNX), the facial nerve (CN7), and the glossopharyngeal (CNIX).

225
Q

225 -Which of the following topical antibacterial agents demonstrates activity against Pseudomonas species?

A. Mupirocin

B. Erythromycin
C. Polymyxin
D. Bacitracin
E. Clindamycin

A

►C

Polymyxins are peptides produced by the organism Bacillus polymyxa. They are bactericidal against Pseudomonas aeruginosa, Proteus mirabilis, Enterobacter, Klebsiella and Escherichia coli. Since they provide little coverage against gram-positive organisms, polymyxins are often combined with other antibacterial agents to increase their spectrum of activity.
Mupirocin covers gram positive organisms mostly, including MRSA, but many reports of resistance are emerging. Topical bacitracin and clindamycin also cover almost exclusively gram positive organisms. Erythromycin has some gram negative (less than gram positive) organism
coverage, but does not reliably treat pseudomonas.

226
Q

226- Which of the following lasers uses a solid as its medium?
A. Argon laser
B. pulsed dye laser
C. alexandrite
D. krypton
E. carbon dioxide

A

►C

Solid media lasers include alexandrite, KTP, ruby, Nd Yag, diode. Liquid media is found in the pulsed dye laser. Gas media is used in argon, carbon dioxide, krypton, and cooper vapor laser.

227
Q

227- The use of EMLA cream is contraindicated in patients with which of the following?

A. Atopic dermatitis
B. Deomycin allergy

C. Sickle cell anemia
D. Methemoglobinemia
E. Peripheral neuropathy

A

►D

The most serious adverse effect of eutectic lidocaine and prilocaine (EMLA) is methemaglobinemia. A metabolite of prilocaine oxidizes hemoglobin to methemoglobin, which is less efficient in release of oxygen leading to tissue hypoxia. Patients with congenital or idiopathic methemoglobinemia or infants under 1 year of age are at higher risk for the development of this side effect.

228
Q

228- Which of the following has been implicated in the promotion of skin aging?

A. alpha-tocopherol
B. homocysteine
C. proanthocyanidin
D. beta carotene
E. alpha-lipoic acid

A

►B

Homocysteine is a sulfhydryl-containing amino acid produced during the conversion of methionine to cysteine. Homocysteine can promote defects in fibrillin-1 and stimulate matrix metalloproteinase production, leading to collagen and elastic fiber degradation. Homocystinuria, caused by a deficiency in cystathionine beta-synthase, presents with osteoporosis, thin skin, and striae. Alpha- tocopherol, beta carotene, proacanthocyanidin, and alpha-lipoic acid have all be described to slow the aging process of the skin via multiple mechanisms including decreasing the inflammatory response and reducing free radical damage.

229
Q

229- Which of the following parameters determines the wavelength of a laser?

A. Medium

B. Fluence
C. Spot size
D. Pulse duration
E. Lens length

A

►A

Laser light is monochromatic light that is emitted at a single wavelength. The wavelength of the laser is determined by the medium in the optical cavity of the laser through which the light passes. The medium may be solid, liquid or gas. Fluence and pulse duration affect the energy delivered by a laser. Spot size influences depth of penetration. Lens length does not influence the wavelength of a laser.

230
Q

230 -Which of the following lasers may cause milia formation as a post-procedure complication?

A. Excimer
B. KTP
C. Pulsed Dye
D. ND-Yag
E. Erbium

A

►E

The erbium (2940 nm) and carbon dioxide (10,600 nm) lasers may both cause milia formation after laser skin resurfacing.

231
Q

231- Which of the following cosmetic injectables can be seen on routine dental x-rays?

A. Botox
B. Zyplast
C. Radiesse
D. Cosmoderm
E. Restylane

A

►C

Radiesse is an injectable, biodegradable filler which is composed of calcium hydroxylapatite microspheres. Calcium hydroxylapatite is a normal constituent of bone and thus can be seen on radiographic imaging. Botox is botulinum toxin; Zyplast is bovine collagen; Cosmoderm is a non- animal form of collagen; and Restylane is a non-animal form of hyaluronic acid.

232
Q

232- A Z- plasty is performed to:

A. Reorient a scar
B. Shorten the length of an excision
C. Close a wound on a convex surface
D. Obtain wound eversion
E. Make use of skin from an area of laxity

A

►A

A Z-plasty is a type of transposition flap that may be used to reorient a scar, especially when the scar crosses skin tension lines. It is also used to change the scar length or vector and ease scar contractures.

233
Q

233- A history of anaphylaxis is a contraindication to treatment with which cosmetic injectable?

A. Sculptra
B. Myobloc
C. Radiance
D. Cosmoderm
E. Restylane

A

►E

Restylane is a hyaluronic acid gel produced by the Streptococcus species of bacteria. The package insert for Restylane warns that its use is contraindicated in patients with “severe allergies manifested by a history of anaphylaxis or the presence of multiple severe allergies”.

234
Q

234- For high risk surgical patients, when is prophylaxis for infective endocarditis ideally first given?

A. 2 days prior to the procedure
B. 1 day prior to the procedure
C. 1 week prior to the procedure
D. 60 minutes prior to the procedure
E. after the procedure

A

►D

Antibiotics are usually given 60 minutes prior to the procedure. The American Heart Association advises it can be given up to 2 hours after the procedure if not given before. Infections are usually polymicrobial, but staph and Strep are of prime concern.

235
Q

235- All of the following are true statements regarding the immunohistochemical stain Melan-A except:

A. Antigen present in 80-100% of melanomas
B. Recognized by CD8+ T cells
C. Sensitive marker for melanocytic nevi
D. Effective in frozen and paraffin-embedded tissue
E. Reliably stains desmoplastic melanoma

A

►E

E Melan-A is a 22-kDa cytoplasmic melanosome-associated glycoprotein also known as MART1 (melanoma antigen recognized by T cells). It is a sensitive marker for both melanocytic nevi and melanoma and may be used on frozen tissue sections as well as paraffin-embedded tissue. Although

the antigen is present in > 80% of melanomas, it does not reliably stain desmoplastic or spindle cell melanomas.

236
Q

236 -Endocarditis prophylaxis is recommended for which

A. Rheumatic heart disease
B. Atrial septal defects
C. Hypertrophic cardiomyopathy
D. Mitral valve prolapse with regurgitation
E. Previous bacterial endocarditis

A

►E

Preoperative antibiotics are recommended for endocarditits prophylaxis in select high risk patients that undergo high risk procedures. High risk procedures are procedures that involve manipulation of gingival tissue or the periapical region of teeth or perforation of the oral mucosa.

High risk patients are, according to the guidelines, those with the following: Prior infective endocarditis, Prosthetic cardiac valves, Unrepaired cyanotic congenital heart defects, including palliative shunts and conduits Congenital heart defects completely repaired with prosthetic material or a device, whether placed by surgery or by catheter intervention, during the first 6 months after the procedure, Repaired congenital defects with residual defects at the site or adjacent to the site of a prosthetic patch or prosthetic device, Cardiac transplants and
development of cardiac valvulopathy.

Patient groups that may have received routine antibiotic prophylaxis in the past but are no longer candidates for it include those with mitral and aortic valve disease, rheumatic heart disease, or structural disorders like ventricular or atrial septal defects or hypertrophic cardiomyopathy, according to the AHA statement.

The revised guidelines were developed with the participation of and have been endorsed by the American Dental Association, the Infectious Diseases Society of America, and the American Academy of Pediatrics. Classically 1 gram of Dicloxacillin or cephalexin is given 1 hour preoperatively and an additional 500 mg is given 6 hours post op. Clindamycin can be givenin those patients who are penicillin allergic.

237
Q

237 -Which of the following medicines has a newly reported side effect of eruptive epidermoid cysts?

A. Minocycline
B. Retinoids
C. Imiquimod
D. Denileukin difitox
E. Potassium iodide

A

►C

Imiquimod is a topical immunomodulator which induces the toll like receptor 7 and is used in the treatment of genital warts, actinic keratoses, and superficial basal cell carcinomas. It has a newly reported side effect of eruptive epidermoid cysts. Minocycline has side effects including a lupuslike syndrome and hepatitis. Retinoids has numerous side effects including hypertriglyceridemia, elevated liver enzymes, dryness of skin, teratogenicity, and most recently inflammatory bowel disease. Denileukin difitox, an engineered protein combining interleukin-2 and Diphtheria toxin, used in mycosis fungoides may cause vascular leak syndrome characterized by hypotension, edema, and pleural effusion. Potassium iodide may lead to the Wolff-Chaikoff effect in which thyroid hormone synthesis is inhibited from excess iodides that block organic iodides from binding in the thyroid.

238
Q

238 -The role of imiquimod in the treatment of superficial basal cell carcinoma is based on the induction of which cytokine?

A. Interleukin-2
B. Tumor growth factor-alpha
C. Interleukin-4
D. Interferon-gamma
E. Tumor necrosis factor-beta

A

►D

Imiquimod is an immunomodulator which induces the production of cytokines that are involved in cell-mediated immunity. These cytokines include interferon-alpha, interferon-gamma, interleukin-1,

interleukin-10 and tumor necrosis factor-alpha. Induction of these cytokines results in antiviral and antitumor activity in vivo.

239
Q

239- The O to Z flap is what type of flap?

A. Rotation
B. Advancement
C. Transposition
D. Pedicle
E. Bilobed

A

►A

The O to Z flap is a bilateral rotation flap in which tissue is moved from two ends to cover a central defect. It is often used on the scalp or lower extremities in locations where tissue laxity is not present.

240
Q

240 -The pain associated with Botulinum A Exotoxin injection is attributed to the:

A. Needle gauge
B. PH
C. Preservative-free saline
D. Exotoxin
E. Temperature

A

►C

Botulinum A exotoxin is used for multiple reasons, most often for the treatment of dynamic facial lines. In a double-blind, randomized controlled study, investigators found that botulinum A exotoxin reconstituted with preservative-containing saline less painful than with preservativefree saline.

241
Q

241 -This muscle overlies the nasal bone and attaches to the nasal root and is also responsible for the “bunny lines” of the nose:

A. Procerus
B. Nasalis
C. Obicularis oculi
D. Zygomatic major
E. Zygomatic minor

A

►B

The nasalis muscle is responsible for the foreshortening of the nose. It is responsible for the ‘bunny lines.”

242
Q

242 -What risk does an organ transplant recipient have of developing melanoma?

A. Same risk as general population
B. Less risk than general population
C. 3-fold increased risk
D. 10- fold increased risk
E. 50-fold increased risk

A

►C

Organ transplant recipients are at risk for developing many cutaneous tumors. The risk of melanoma is 3-fold that of the general population whereas these patients have a 65-fold increased risk of developing squamous cell carcinoma.

243
Q

243 -The main advantage of selecting 4-0 Vicryl rather than 5-0 Vicryl to suture a wound is:

A. Smaller suture diameter
B. Increased tensile strength
C. Increased knot security
D. Smaller needle
E. Increased suture memory
c

A

►B

Sutures are classified according to the United States Pharmacopeia (USP) criteria. This classification system specifies the diameter of a given suture material that is required to produce a certain tensile strength. The smaller the cross-sectional diameter of a suture material, the higher the USP number that is assigned. Thus, 4-0 Vicryl will have greater tensile strength and a larger cross-sectional diameter than 5-0 Vicryl.

244
Q

244 -An M-plasty is performed to:

A. Decrease tension on a wound
B. Obtain wound eversion
C. Shorten the length of an excision
D. Reorient a scar
E. Make use of excess tissue

A

►C

An M-plasty is a variation of the fusiform excision where either one or both ends of the ellipse are modified. The main advantages to performing an M-plasty are to shorten the length of an excision and to correct dog-ears.

245
Q

245- Which anesthetic has the shortest duration of action?

A. Mepivacaine

B. Procaine
C. Bupivacaine
D. Tetracaine
E. Prilocaine

A

►B

Procaine has the shortest duration of action, only lasting about 30-60 minutes. It is an ester anesthetic and is commonly used in dentistry.

246
Q

246 -This agent should not be used in pregnant patients and in children because it can be teratogenic:

A. Hexachlorophene
B. Chlorhexidine
C. Providone iodine
D. Bentadine
E. Hibiclens

A

►A

Hexachlorophene should not be used in children or in pregnant women. It has a potential for neurotoxicity and can be teratogenic.

247
Q

247- Which one of the following lasers would be effective in the treatment of rhinophyma?
A. Pulsed dye laser
B. Ruby laser
C. Alexandrite laser
D. KTP laser
E. Carbon dioxide laser

A

►E

The carbon dioxide laser is a 10,600 nm laser that can be used to treat rhinophyma. The advantage of the carbon dioxide laser over steel or dermabrasion is that the laser is relatively bloodless. The erbium YAG can also be used to treat rhinophyma.

248
Q

248- Which tattoo pigment is most commonly associated with allergic reactions, eczematous and granulomatous?

A. Mercuric sulfide
B. Titanium dioxide
C. Cadmium sulfide
D. Cobalt
E. Carbon

A

►A

Allergic reactions have been reported with several different types of tattoo pigment. The most commonly associated tattoo pigment however is mercuric sulfide. Tattoo with mercuric sulfide produces a red color.

249
Q

249- When would use of this technique be most beneficial?

A. To prevent post-operative bleeding
B. To prevent post-operative infection
C. To shorten the length of the wound
D. To increase the width of the wound
E. To prevent pincushioning

A

►C

M-plasty is used to shorten wound length and is often when it is not desirable to extend the ellipse into a nearby structure.
This becomes important when reconstructive efforts lie adjacent to important nearby free margins or cosmetic subunit junctions.
M-plasty repair does not decrease the risk of post-operative bleeding or infection, nor does it
lengthen the wound or decrease pincushioning in any meaningful way

250
Q

250- Damage to the zygomatic branch of the facial nerve leads to which of the following:

A. Corner of the mouth droop
B. Inability to pucker lips
C. Eyelid ectropion and inability to close eyelid
D. Unilateral eyelid ptosis
E. Winged scapula

A

►C

Damage to the zygomatic nerve causes eyelid ectropion and inability to close eyelid.

251
Q

251- A Mohs surgery fellow experiences burning and tingling of her fingertips 6 months after starting her fellowship. She most likely has been exposed to which one of the following chemicals?

A. Chlorhexidine gluconate
B. Hematoxylin
C. Povodine-iodine
D. Hexachlorophene
E. Toluidine blue

A

►D

Hexachlorophene is an antibacterial agent that was first introduced in 1944. Its use was discontinued in the 1970s when it was discovered to have neurotoxic side effects.

252
Q

252 -Which of the following is a complication of Thermage treatment?

A. Keloid formation
B. Lipoatrophy
C. Granuloma formation
D. Hyperpigmentation
E. Herpes simplex infection

A

►B

Thermage (ThermaCool) is a nonablative radiofrequency-based system which uses volumetric heating to induce tightening of the skin and dermal remodeling. Lipoatrophy is a late -occurring potential complication of this technology.

253
Q

253- Which of the following surgical prepatory solutions is teratogenic?

A. Povidine-iodine
B. Chlorhexidine
C. Ethyl and Isopropyl alcohol
D. Hexachlorophene
E. Benzalkonium chloride

A

►D

Hexachlorophene or pHisoHex has shown to be toxic to developing embryos in animal studies and thus is prohibited for use in pregnant females. Chlorhexidine is a common ingredient in oral rinses(Peridex) and surgical prep solutions(Hibiclens) can cause ototoxicity if in contact with the middle ear and ocular toxicity if in contact with the eyes. Ethyl alcohol is effective against both gram positive and gram negative organisms. Both Povidine-Iodine (Betadine)and Benzalkonium chloride (Zephiran) can cause an allergic contact dermatitis.

254
Q

254 -The cutaneous lip and chin are divided into cosmetic units by which anatomic boundary?

A. Vermillion border
B. Nasolabial fold
C. Marionette lines
D. Mental crease
E. Philtral ridge

A

►D

The term cosmetic unit is used to define regions of the face by specific characteristics such as color, texture, presence of hair, and prominence of sebaceous glands. In dermatologic surgery, it is preferable to conceal incisions within the boundary lines of cosmetic units. The mental crease divides the cutaneous lip and chin.

255
Q

255 -The tensile strength of a wound 6 months after surgery compared to intact skin is:

A. 15%
B. 30%
C. 50%
D. 70%
E. 100%

A

►D

The tensile strength of a wound approaches 70% of normal skin strength at 6 months postoperatively. Wound healing is a process which takes many months, yet the tensile strength of a wound never exceeds 80% of the tensile strength of intact skin. It is therefore critical to use long- lasting subcuticular sutures to minimize the tension on a healing wound and prevent scar widening.

256
Q

256 -How long after cutaneous infiltration of lidocaine with epinephrine is maximal vasoconstriction achieved?

A. 1 minute
B. 7 minutes
C. 15 minutes
D. 30 minutes
E. 1 hour

A

►C

The use of epinephrine with local anesthesia has two main advantages. Firstly, the epinephrine is a vasoconstrictor and assists in controlling bleeding during surgery. Secondly, and also a direct result of its vasoconstrictive effects, epinephrine helps prolong the duration of the anesthetic agent 100% to 200% by delaying its absorption from the surgical site. Although the anesthetic properties of lidocaine take effect within the first few minutes of infiltration, the vasoconstrictive effects of epinephrine require approximately 15 minutes to fully develop.

257
Q

257- All of the following statements are true regarding the „MCW Melanoma Cocktail‟ except:

A. it is a mixture of monoclonal antibodies to MART-1
B. it is performed intraoperatively
C. it is a mixture of monoclonal antibodies to Melan-A
D. it demonstrates micrometastases in sentinel lymph nodes
E. it is a mixture of polyclonal antibodies to tyrosinase

A

►E

The „MCW Melanoma Cocktail‟ is an immunostain made up of monoclonal antibodies to MART-1, Melan-A and tryrosine. It is used intraoperatively during sentinel lymph node biopsy and allows for rapid and accurate determination of micrometastases. Shidham VB et al. Optimization of an immunostaining protocol for the rapid intraoperative evaluation of melanoma sentinel lymph node imprint smears with the „MCW Melanoma Cocktail‟. Cytojournal 2004;1(1):2.

258
Q

258 -Which cosmetic filler substance is contraindicated in the glabella?

A. Bovine collagen
B. Human collagen
C. Hyaluronic acid
D. Calcium hydroxyapatite
E. Poly-L-lactic acid

A

►A

As placement of bovine collagen is typically deeper in the tissues compared to other filler, its use has resulted in vascular occlusion and necrosis in the glabellar area, and is therefore contraindicated.

259
Q

259- Which of the following cosmetic injectables has the longest duration of action?

A. Botox
B. Sculptra (poly L-lactic acid)
C. Restylane (hyaluronic acid)
D. Cosmoderm (collagen)
E. Myobloc

A

►B

Sculptra (called New-Fill outside of the US) is a biodegradable filler composed of poly-L-lactic acid, the same material used in absorbable sutures. Preliminary studies have demonstrated longevity of the filler at two years post-treatment. Valantin MA et al. Polylactic acid implants (New-Fill) to correct facial lipoatrophy in HIV-infected patients: results of the open-label study VEGA. AIDS. 2003;17:2471-2477.

260
Q

260- What is the ratio of sodium bicarbonate to 1% lidocaine in a buffered lidocaine solution?

A. 1:1
B. 1:2
C. 1:5

D. 1:10
E. 1:25

A

►D

The use of sodium bicarbonate in 1% lidocaine reduces the acidity of the local anesthetic. Unbuffered lidocaine has pH of approximately 5.5 -7.0 and lidocaine with epinephrine has a pH of approximately 3.5 - 5.0. The addition of sodium bicarbonate at a ratio 1:10 raises the pH to a more physiologic level, thereby reducing the stinging sensation associated with injection. However, the addition of bicarbonate also reduces the shelf-life of the lidocaine.

261
Q

261- Which of the following local anesthetic agents should not be used in children?

A. Bupivicaine
B. Etidocaine
C. Mepivicaine
D. Prilocaine
E. Benzocaine

A

►D

P r i l o c a i n e s h o u l d n ’ t b e u s e d i n c h i l d r e n g i v e n t h e r i s k o f methemoglobinemia.Methemoglobulinemia in children can occur from exposure to oxidizing substances such as aniline dyes, prilocaine or pyrimidine. In methemoglobinemia, the iron in hemoglobin is oxidized from the ferrous state (Fe 2+) to ferric (Fe3+) resulting in the inability to transport oxygen and carbon dioxide. Clinically, this condition results in cyanosis.

262
Q

262- Treatment with which of the following lasers has been effective in psoriasis?

A. Carbon dioxide laser
B. Excimer laser
C. Ruby Laser
D. Pulsed dye laser

E. Alexandrite laser

A

►B

The excimer laser is a 308 nm wavelength laser that has been used to treat psoriasis. The excimer laser has some advantages over ultraviolet therapy. By treating only involved skin, higher doses can be used and clearance may occur with fewer treatments.

263
Q

263- Which of the following is a vasoconstrictor in the absence of epinephrine?

A. lidocaine
B. procaine
C. cocaine
D. bupivacaine
E. prilocaine

A

►C

Cocaine is the only anesthetic that vasoconstricts in the absence of epinephrine. Cocaine is an ester anesthetic. Full vasoconstriction effects of epinephrine requires 7 -15 minutes. Epinephrine is pregnancy category C.

264
Q

264- Which material is used to coat the jaws of the needle holder in the photograph?

A. Gold
B. Tungsten carbide
C. Copper
D. Stainless steel
E. Nickel

A

►B

Tungsten carbide is believed to be a harder more durable material that allows for better grasping of the needle. Needle holders with tungsten carbide jaws usually have gold-plated handles.

265
Q

265- What is the tattoo pigment responsible for most lichenoid reactions?

A. Titanium dioxide
B. Carbon
C. Mercuric sulfide
D. Chromates
E. Iron oxide

A

►C

Lichenoid tattoo reactions are not as common as eczematous reactions, both of which are most commonly caused by mercuric sulfide. Lichenoid reaction are likely to be mediated by delayed hypersensitivity to a lymphocytic T-cell infiltrate.

266
Q

266- Which one of the substances is not a component of a Jessner’s peel?

A. Ethanol
B. Salicylic acid
C. Lactic acid
D. Phenol
E. Resorcinol

A

►D

Jessner’s is a light-medium chemical peel that contains salicylic acid, lactic acid, ethanol, and resorcinol. Phenol is used in deep peels, ie the Baker-Gordon peel which also contains croton oil, water, and septisol. Phenol may induce cardiac arrythmias and liver or renal failure; hydration i s important in all phase of treatment.

267
Q

267- Which of the following cosmetic injectables binds water to create volume, has a low allergic reaction profile, and lasts 6 to 12 months?

A. Botulinum toxin
B. Calcium hydroxylapatite
C. Human-derived collagen
D. Hyaluronic acid
E. Silicone

A

►D

Hyaluronic acid is a natural component of human connective tissue. As a cosmetic filler, hyaluronic acid is an attractive alternative to currently available filler substances: its ability to bind water creates volume and plumps the skin; its duration of action of approximately 6-12 months is appreciably longer than conventional collagen; and since hyaluronic acid is chemically identical across all species, hypersensitivity reactions are rare.

268
Q

268- The use of EMLA cream is contraindicated in patients with which of the following?
A. Atopic dermatitis
B. Neomycin allergy
C. Sickle cell anemia
D. Methemoglobinemia
E. Peripheral neuropathy

A

►D

EMLA is a topical anesthetic composed of a eutectic mixture of 2.5% lidocaine and 2.5% prilocaine. The major concern when using EMLA is the potential risk of methemoglobinemia. Patients with glucose-6-phosphate deficiency and patients taking methemoglobin-inducing agents (dapsone, phenytoin, sulfonamides) are more susceptible to developing methemoglobinemia. In addition, care should be taken when using EMLA in infants less than 3 months of age because of the incomplete maturation of their NADH-methemoglobinemia reductase system.

269
Q

269- Which part of the eye is most likely to be damaged to exposure to a pulsed dye laser?

A. Retina
B. Cornea
C. Lens
D. Conjuctiva
E. Sclera

A

►A

The pulsed dye laser operates at 585-nm and targets hemoglobin as a chromophore. It can pass through the cornea and damage choroidal and retinal vasculature. Several pigment-specific lasers (eg, ruby, alexandrite, Nd:YAG) may also damage the retina by targeting the retinal pigment. The eye damage is related to the chromophore of the laser. The lasers that target water damage the cornea.

270
Q

270- The repair demonstrated in the photograph is a:

A. Split-thickness skin graft
B. Interpolation flap
C. Full-thickness skin graft
D. Rotation flap
E. Transposition flap

A

►B

The postauricular flap is useful for repairing extensive defects of the helical rim, particularly when cartilage is involved. It is an example of an interpolation flap.

271
Q

271- Epinephrine should be avoided with anesthetics in which of the following conditions?

A. Hypothyroidism
B. Diabetes Mellitus
C. Renal failure
D. Liver failure
E. Pheochromocytoma

A

►E

Epinephrine should be avoided in patients with a pheochromocytoma.

Explanation: Absolute contraindications to epinephrine use include pheochromocytoma and hyperthyroidism. Patients with poorly controlled hypertension and severe cardiovascular disease, as well as those who are pregnant, should also be considered with caution prior to using
epinephrine. Use in the periorbital area in patients with a history of narrow angle glaucoma should
be avoided as well. Liver failure, renal failure, diabetes mellitus, and hypothyroidism are not contraindications to epinephrine use with anesthesia.

272
Q

272- The type of repair depicted in the photograph is a:

A. Rhombic flap
B. Nasolabial flap

C. Rotation flap
D. Full-thickness skin graft
E. Island pedicle flap

A

►A

The rhombic flap is a type of transposition flap. The classic rhombic flap is designed with two 60 degree angles and two 120 degree angles. The point of maximum tension is at the closure of the donor site.

273
Q

273 -The following are true statements regarding the design of an excisional ellipse except:

A. Ellipse includes pathologic and normal tissue
B. Length of ellipse is at least 3 times its total width
C. The angle at each of the apices is 15º
D. Incision lines are placed in relaxed skin tension lines
E. Long axis of ellipse is oriented perpendicular to free margins

A

►C

Thoughtful design of an elliptical excision is important for obtaining optimum tissue diagnosis, tumor clearance, wound closure and cosmetic result. The above statements are all true except that the angles at the apices of the ellipse should be 30º.
A properly designed ellipse should be at least three times as long as it is wide, include the lesion of interest with proper margins, and have 30 degree apical angles. Repair of surgical defects should always attempt to: 1) maintain function 2) preserve free margins 3) respect cosmetic subunit
boundaries 4) hide scar in relaxed skin tension lines.

274
Q

274- All of the following thrombotic complications have been reported after discontinuing aspirin therapy prior to surgery except:

A. Stroke

B. Myocardial infarction
C. Pulmonary embolism
D. Transient ischemic attack
E. Cerebral embolism

A

►C

Kovich et al. JAAD 2003 evaluated thrombotic complications in patients who had stopped either coumadin or aspirin peri-operatively. Thrombotic complications in patients who stopped warfarin included stroke, TIA, myocardial infarction, cerebral embolism, death, DVT, pulmonary embolus, and blindness. Thrombotic complications in patients who stopped aspirin included stroke, TIA, myocardial infarction, cerebral embolism and death. No DVT or pulmonary embolus was reported.

275
Q

275 -A 60-year-old man receives cephalexin prophylaxis 60 minutes prior to his Mohs surgery for a BCC on the nose. Which condition of his most likely necessitated prophylaxis?

A. History of CABG
B. Secundum atrial septal defect
C. Mitral valve prolapse with regurgitation
D. History of rheumatic fever without valve dysfunction
E. Pacemaker

A

►C

Prior to Mohs patients who are at high risk should receive prophylaxis int he form of cephalexin (or clindamycin or azithromycin if penicillin-allergic) 30-60 minutes prior to the procedures for non- oral skin. High risk patients include those with prosthetic valves, a history of bacterial endocarditis, mitral valve prolapse with regurgitation, mitral valve prolapse without regurgitation in MEN > 45 years, any valve dysfuntion, cardiac malformation, hypertrophic cardiomyoopathy, CNS shunts, and more. Some low-risk examples (no prophylaxis) include s/p CABG, pacemaker, murmur, or mitral valve prolapse without regurgitation.

276
Q

276 -The ultraviolet radiation that is responsible for photoallergic reactions is:

A. UVA
B. UVB
C. UVC
D. UVD
E. Board UV

A

►A

UVA is known to cause photoallergic reactions. It penetrates glass and goes deep into the dermis and can cause immediate erythema.

277
Q

277 -The most common adverse reaction seen with betadine is:

A. Ototoxity
B. Teratogenicity
C. Allergic contact dermatitis
D. Seizures
E. Corneal damage

A

►C

Most common side effect seen with betadine is allergic contact dermatitis, secondary to the iodine component.

278
Q

278 -A patient with a large nasal tip defect had reconstruction of his wound with a forehead flap. After what postoperative interval should the patient schedule the inset of this flap?

A. 1 week
B. 3 weeks
C. 2 months
D. 3 months

E. 6 months

A

►B

The forehead flap is often the reconstructive option of choice for large nasal defects. The flap has a vertically-oriented blood supply which is supplied primarily from the supratrochlear arteries.
Pedicle division and inset of this flap are typically performed 3 weeks after the initial surgery.

279
Q

279 -A patient with a squamous cell carcinoma of the helical rim had excisional surgery followed by closure of the wound with an advancement flap. Which would be the most appropriate antibiotic for this patient to receive postoperatively?

A. Dicloxacillin
B. Cephalexin
C. Erythromycin
D. Ciprofloxacin
E. Trimethoprim-sulfamethoxazole

A

►D

Infection after cutaneous surgery is typically due to Staphylococcal and Streptococcal organisms. However, Pseudomonas aeruginosa may be normal flora of the ear and thus can potentially complicate ear surgery by causing severe infection of the external ear canal (malignant otitis externa). Ciprofloxacin would be the antibiotic of choice for this patient because it is effective against Pseudomonas species.

280
Q

280 -Which of the following determines the wavelength of a laser?

A. Fluence
B. Q switch
C. Medium
D. Spot size
E. Pulse duration

A

►C

The wavelength of a laser is determined by the medium.

281
Q

281 -Treatment with which modality would be most effective for the patient shown?

A. Pulsed dye laser
B. Intense pulsed light
C. Photodynamic therapy
D. Radiofrequency
E. Erbium:YAG laser

A

►B

The patient shown has multiple solar lentigines. Targeting the melanosomes in these benign, superficial pigmented lesions would be most successfully accomplished with the intense pulsed light source.

282
Q

Surgical scissors (left to right): O’Brien, straight suture, Gradle.

A
283
Q

Surgical scissors (left to right): Westcott, iris (tungsten carbide), baby Metzenbaum, Metzenbaum.

A
284
Q

Bandage scissor

A
285
Q

Needle holders (left to right): Halsey, MayoHegar, Castroviejo

A
286
Q

calpel handles (left to right): Round knurled handle, Beaver blade handle, #3 handle.

A
287
Q

Blades used in dermatologic surgery (left to right): #10, #15, #15c, #11.

A
288
Q

Forceps (left to right): Jeweler’s forceps, Adson forceps without teeth, Adson forceps with teeth, BishopHarmon forceps.

A
289
Q

Nail surgery instruments (top to bottom): Nail elevator and English nail splitter.

A
290
Q

Single- and double-pronged skin hooks.

A
291
Q

Fox dermal curette.

A
292
Q

Chalazion clamp

A
293
Q

Mosquito hemostat

A